Critical Reasoning
Mixed Assignment
Questions 3
Answers 74
Questions
1. Louis: The financial industry will suffer greatly from the new government
regulations over derivatives. Compliance with these regulations will cost the
industry $5.5 million annually because companies in the industry will have to
double the size of their proxy statements in order to disclose all the required
information. Companies in the industry will then lose profits and have to lay off
other employees. Therefore, these regulations will have an adverse effect on
the nation’s economy.
Peter: The $5.5 million that companies in the finance industry will have to
spend will be profits for other types of companies in the private sector. Profits
and jobs may be lost in the finance industry, but they will be gained by other
companies.
Peter responds to Louis by
A. agreeing with Louis’s conclusion by offering additional
information to support it.
B. offering information that suggests that Louis has overlooked a
mitigating consequence.
C. agreeing with Louis’s conclusion but suggesting that the
outcome is positive rather than negative.
D. challenging the tenability of the facts that serve as the basis of
Louis’s argument.
E. demonstrating that Louis’s conclusion is not based on relevant
facts.
2. A cellular phone company spent tens of thousands of dollars in the
past year fixing cell towers damaged by extreme weather conditions. In
order to reduce the company’s overall budget this year, the company
plans to cover the base of its towers with a better type of steel and to
move certain towers to different locations so that fewer overall are
needed.
Which of the following would NOT be required for the cellular
phone company to achieve its aim?
A. The cost of the steel would have to be lower than the cost of
the repairs to the cell towers during the previous year.
B. The cost of evaluating the most efficient placing of cell towers
would have to be lower than the cost of the repairs to the cell
towers during the previous year.
C. The cost of covering the towers with the better type of steel
would have to be lower than the cost of the repairs to the cell
towers during the previous year.
D. The cost of the repairs to the cell towers during the previous
year would have to be lower than the price of the steel
construction crews’ wages to cover the cell tower bases.
E. The cost of shipping the steel would have to be lower than the
cost of the repairs to the cell towers during the previous year.
3. Kathryn has a higher GPA than Pamela. Madelyn has a higher GPA
than Adrienne. Shane has a higher GPA than Madelyn. Thus, it
follows that Kathryn has a higher GPA than Adrienne.
Which of the following, if introduced into the argument as an additional
premise, makes the argument above logically incorrect?
A. Madelyn has a higher GPA than Katherine.
B. Katherine has a higher GPA than Madelyn.
C. Adrienne has a higher GPA than Pamela.
D. Pamela has a higher GPA than Madelyn.
E. Adrienne has a higher GPA than Shane.
4. The Braman Rule states that someone who leaves property to
another in a will cannot leave property that the deceased did not own at
the time of death. Humphrey Monroe left Claire McGinnis a cottage in
his will. Claire McGinnis stated in her will that all her property would be
left to her niece, Sharon. Claire McGinnis died before Humphrey
Monroe. Therefore, the cottage was not inherited by the person whom
Claire McGinnis wanted to inherit it.
Which of the following assumptions would make the conclusion
above properly drawn?
A. Claire McGinnis wanted Sharon to inherit the cottage.
B. Humphrey Monroe wanted Sharon to inherit the cottage.
C. Sharon wanted to inherit the cottage.
D. Claire McGinnis did not want Sharon to inherit the cottage.
E. Humphrey Monroe did not want Sharon to inherit the
cottage.
5. A franchise restaurant pays its employees in State A $2.00 per hour
above the minimum wage, and pays its employees in State B $1.00 per
hour above the minimum wage. The franchise has three times as many
employees in State A as in State B. The franchise’s total expenditures on
hourly wages for its employees must therefore be higher in State A than
in State B.
The argument above relies on which of the following
assumptions?
A. The franchise employees in State A do not each work
significantly less hours than the franchise employees in State B.
B. The franchise has a greater number of restaurants in State A
than in State B.
C. The franchise employees in State B each work significantly
more hours than the franchise employees in State A.
D. The law in State A requires the franchise to pay its employees a
higher wage than the law in State B.
E. The franchise is planning to decrease the wages paid to
employees in State A to $1.00 per hour above the minimum
wage.
6. The cost of buying diamonds in the country of Luvania is 40% more
than the cost of buying diamonds in the country of Oretania. When one
adds the expenses of travel and the tax on purchases by foreigners in
Oretania, buying diamonds in Oretania is still less expensive for a
citizen of Luvania than buying diamonds in Luvania.
The statements above, if true, best support which of the following as a
conclusion?
A. The ground in Oretania breaks more easily under the force of a
pickaxe than it does in Luvania.
B. The tax on purchases by foreigners in Oretania is less than 40% of
the cost of buying diamonds in Luvania.
C. The cost of traveling to Oretania from Luvania is more than the
40% of the cost of buying diamonds in Luvania.
D. Diamond mining costs are higher in Luvania than in Oretania.
E. Buying diamonds in Luvania will eliminate some retail jobs in
jewelry stores in Oretania.
7. After reports from three schools of too much chlorine in their
drinking water, the city hired a plumbing company to revamp its
running water system. The plumbing company replaced the filtration
system after the point in which groundwater is collected. We can
conclude that the three schools will no longer have a problem with too
much chlorine in their drinking water.
Which of the following does the argument depend on as an
assumption?
A. Not all faulty water filtration systems will allow too much
chlorine into the drinking water.
B. Water filtration systems are not the only source of too much
chlorine in the drinking water.
C. The city has been able to successfully filter water at its water
filtration plant.
D. The excess chlorine was not introduced into the schools’ water
supply at some point later than groundwater collection.
E. Not all faulty water filtration systems are likely to produce excess
chlorine in a drinking water supply.
8. The number of people who quit smoking every year is on the rise. The
most common reasons that people cited in a survey for quitting smoking
are wanting to improve their health, wanting to save money, and
wanting to appear physically appealing. Out of the people who quit
smoking that cited health reasons, 85% of them also cited wanting to
appear physically appealing. The last question in the survey asked
whether physical appearance was important to the respondents. 100%
of survey respondents answered affirmatively.
Which of the following, if true, does NOT explain the discrepancy
described above?
A. Some respondents felt that although their physical appearance
was important to them, it was not important enough to quit
smoking.
B. Some respondents believed that holding a cigarette enhanced
their physical appearance.
C. Some respondents believed that cigarette smoking would not
have an adverse effect on their physical appearance.
D. Some respondents felt superficial in admitting that their physical
appearance was important to them.
E. Some respondents believed that cigarette smoking had little to
no connection to their physical appearance.
9. Rental prices are usually lower on the basement and ground floors of
an apartment building, and highest at the top of the building. Rental
prices are also usually higher for buildings that receive more sunlight.
In addition, rental prices are determined by the number of bedrooms in
an apartment. There are six floors in building #153. Cathy lives on the
sixth floor, David lives on the fourth floor, Krystal lives on the ground
floor, and Oscar lives in the basement. Out of the four tenants, Krystal
pays the highest rent and Oscar pays the lowest rent.
Which of the following conclusions CANNOT be drawn from the
statements above?
A. Oscar lives in a 1-bedroom, David’s apartment receives the
most sunlight, and Krystal lives in a 3-bedroom.
B. Cathy lives in a 3-bedroom, Oscar lives in a 3-bedroom, and
Krystal lives in a 1-bedroom.
C. Krystal lives in a 1-bedroom, David’s apartment receives the
most sunlight, and Cathy lives in a 3-bedroom.
D. David lives in a 2-bedroom, Cathy’s apartment receives the
most sunlight, and Oscar lives in a 1-bedroom.
E. Cathy lives in a 3-bedroom, Cathy’s apartment gets the most
sunlight, and Krystal lives in a 5-bedroom.
10. A university study showed that women who spent more money on
their exercise clothes went to the gym more often. However, the
women who spent more money on their exercise clothes did not lose
more weight than the women who spent less money on their exercise
clothes.
Which of the following statements does NOT help to explain the results
of the study?
A. The women who spent more money on their exercise clothes did
not exercise as hard as those who spent less money because they
were too concerned about looking good at the gym.
B. The women who spent less money on their exercise clothes
worked out more frequently than those who spent more; they
just did not work out as frequently at the gym.
C. The women who spent more money on their exercise clothes
focused their workouts more on strength training and less on fat
and/or calorie burning exercise.
D. The women who spent more money on their exercise clothes
were, on average, already at their ideal weight.
E. The women who spent more money on their exercise clothes had
higher memberships in weight loss programs.
11. In the past, doctors would often prescribe antibiotics to patients
with viruses purely for purposes of patient satisfaction. Antibiotics only
successfully treat bacterial infections and do not actually treat viruses,
which must merely run their course. Doctors have since decreased the
percentage of patients for whom they prescribe antibiotics.
All the following statements help to explain the decrease in the
percentage of patients prescribed antibiotics EXCEPT:
A. Doctors are less concerned with patient satisfaction.
B. Doctors are more concerned about patients developing an
immunity to antibiotics.
C. A larger percentage of patients today are diagnosed with
viruses.
D. A smaller percentage of patients today are diagnosed with
ailments that require antibiotics.
E. A larger percentage of patients today are diagnosed with
bacterial infections.
12. A publishing company derives revenue from three main areas.
Digital products constitute 30% of sales and earn 40% of the profits.
Books constitute 60% of sales and earn 50% of the profits. Audiobooks
and other mediums constitute 10% of sales and earn 10% of the profits.
Which of the following can be inferred from the statements above?
A. Digital products earn higher profits per sale than books or
other mediums.
B. Digital products earn more profit than books or other
mediums.
C. The area of digital products is growing more rapidly than
books.
D. Each digital product is more profitable than each book
product.
E. Books earn less profit than digital products and products in
other mediums combined.
13. Major businesses have begun near-shoring, which is a practice that
involves moving jobs from cities with a higher cost of living to lower
cost of living locales. The rationale behind near-shoring is that
employees with a lower cost of living will not demand as high salaries.
High-level positions that involve a lot of face time with clients will
remain in the higher-cost-of-living cities. High-level employees will
travel to the lower-cost-of-living locales when they need to meet with
staff.
Which of the following conclusions, if true, provides the strongest
support for the practice of near-shoring?
A. Employees will be willing to relocate in order to keep their
jobs.
B. The cost of travel for high-level employees will not negate the
savings in moving jobs to locations with a lower cost of living.
C. Clients will not need to travel to the lower-cost-of-living
locales.
D. Cities with a higher cost of living will not offer financial
incentives to businesses to stay.
E. Clients will not move out of the cities with a higher cost of
living.
14. In order for humanitarian effort to relieve starving that results
from a drought to succeed, there must be an efficient
transportation system for water and support from several relief
organizations. At least a dozen trucks that hold liquid are required
for an efficient transportation for water. Each relief organization
involved has pledged to supply trucks. Thus, relief will be brought to
drought victims in the country of Y.
Which of the following is the most important item be ascertained in
order to evaluate the argument presented above?
A. the number of relief organizations involved
B. the availability of gasoline for the trucks
C. the availability of water for transportation
D. the amount of time it would take for the trucks to transport
the water to drought victims
E. the amount of time it would take to begin transporting the
water to drought victims
15. Alexandra has more money than Courtney. Marnie has more
money than Courtney. Alexandra has less money than Danielle. We
can conclude that Courtney has less money than Josie.
Which of the following, if true, would make the argument above
logically incorrect?
A. Danielle has more money than Marnie.
B. Danielle has less money than Courtney.
C. Alexandra has more money than Josie.
D. Josie has more money than Danielle.
E. Josie has less money than Marnie.
16. Taylor: The requirement that financial institutions register with the
Securities and Exchange Commission all securities offered for sale to
the public is pointless because people never read through all those piles
of paper before they buy securities.
Sophia: I disagree. The registration provides a paper trail that people
can use to hold financial investors responsible in the event that
securities were not accurately represented.
Sophia responds to Taylor’s argument by
A. providing evidence that people who buy securities actually read
the paperwork about them beforehand.
B. shifting the discussion away from the paper that registration
generates and toward the illegal activity that often occurs in
securities sales.
C. pointing out a benefit of registration, which Taylor’s argument
failed to consider.
D. objecting to Taylor’s dismissal of securities purchasers’ financial
literacy.
E. rejecting Taylor’s argument while suggesting that registration
with the Securities and Exchange Commission be required of all
securities.
17. The two methods available to ABC Car Company to increase fuel
efficiency in its vehicles are installing a new type of engine in its cars
and using a lighter metal for the car frames. The use of a lighter metal in
the car frames will increase efficiency more than installation of a new
type of engine. Therefore, by using a lighter metal in the car frames,
ABC Car Company will be doing the most it can to increase fuel
efficiency in its cars.
If the statements above are true, which of the following must be true?
A. ABC Car Company has several options for a lighter metal to
use for the car frames.
B. Fuel efficiency cannot be increased more by using both
methods together than by only using a lighter metal in the car
frames.
C. The cars that ABC Car Company manufactures are already
quite fuel efficient.
D. ABC Car Company is unsure whether it should bother with
increasing fuel efficiency.
E. Installing a new type of engine will be more expensive than
using a lighter metal in the car frames.
18. Demand for olive oil has remained constant during the past few
years. Production and prices have also remained the same during the
past few years, yet Crunchy Organics has seen an increase in its profits
from olive oil by more than 20% over last year’s profits.
The conclusion above would be more reasonably drawn if any of the
following were inserted into the argument as an additional premise
EXCEPT:
A. An economic recession has enabled Crunchy Organics to find
workers who will work for lower wages.
B. Crunchy Organics installed insulation and other energy-saving
measures in its facilities, which resulted in significant decreases
in utility bills and a hefty tax refund.
C. Increased rainfall resulted in a more plentiful olive crop.
D. Crunchy Organics was featured in a nationwide news segment
about ethical small businesses, which was viewed by over 10
million people.
E. The demand for olive oil has decreased, and most companies
that produce olive oil have increased their distribution.
19. Which of the answer choices best completes the argument below?
More cosmetic companies should follow the marketing plan of
Company G. Women want to see how products look on women who
look like themselves because they can get a better idea of how the
makeup will actually look on them.
Women are also tired of being confronted with images ______.
A. of women who can clearly afford more expensive cosmetics than
the average woman.
B. of supermodels who wear evening makeup when most women
shop for a daytime look.
C. of supermodels who perpetuate the stereotype of unrealistic
beauty.
D. of models who fail to make cosmetics look glamorous.
E. of models wearing makeup that is not of a good quality.
20. Township P conducted a survey in which over half of its residents
admitted that they do not recycle, and the township’s recycling plant
has threatened to lay off employees due to a lack of work. In order to
increase recycling, Township P should install recycling machines in the
town center, in which residents can turn in their bottles, cans, and
other recyclables in exchange for five cents apiece. This plan would
dramatically increase the number of residents that recycle and save
the jobs of many residents.
Which of the following are assumptions made in drawing the
conclusion above?
A. Saving the jobs at the township’s recycling plant would not result
in corresponding layoffs at the township’s trash plant, and the
5-cent refund would pay for the cost of recycling the addi tional
materials.
B. The township’s recycling plant can handle the increased quantity
of materials to recycle, and an increase in recycling would save
the jobs that the recycling plant has threatened to eliminate.
C. The recycling machines would induce residents to recycle more,
and the 5-cent refund would pay for the recycling machines over
time.
D. The 5-cent refund per item would eventually pay for the cost of
the recycling machines, and the township could afford to pay the
workers that the recycling plant has threatened to lay off.
E. The recycling machines and the 5-cent refund per item would
induce residents to recycle more, and an increase in recycling
would save the jobs that the recycling plant has threatened to
eliminate.
21. Reporter: The Q Virus is a serious disease that can make people
deathly ill. Starting in 1990 and continuing in every subsequent year,
approximately 10,000 people became infected. Before 1990, there
were only about 3,000 cases per year. We can conclude that the Q
Virus has become a dire public health threat.
Public Health Expert: But before 1990, the population was only one
quarter the size that it is now.
The public health expert challenges the reporter’s argument by doing
which of the following?
A. pointing out that the argument makes a conclusion based on a
small sample of the population
B. pointing out that the argument treats facts about some people
in a group as if they were true about all people in that group
C. presenting information that refutes the argument’s assumption
of an incremental increase
D. introducing information that casts doubt on an assumption of
the reporter’s argument
E. questioning the argument’s information about the people in a
particular group
22. Computer programming classes are offered in the College of Math
and Science. A middle range percentage of students is enrolled in
courses of the College of Math and Science. Economics classes are
offered in the College of Business. The largest percentage of students is
enrolled in courses in the College of Business. Business writing courses
are offered in the College of Humanities. The lowest percentage of
students is enrolled in courses in the College of Humanities. Therefore,
computer information system classes should be offered in the College of
Business.
The conclusion above would be more reasonably drawn if which of
the following were inserted into the argument as an additional
premise?
A. Courses should be offered in the college in which a greater
percentage of students is enrolled.
B. Computer information systems courses are more like business
than they are like computer science.
C. More majors in the College of Business require courses in
computer information systems than majors in the College of
Math and Science.
D. Computer information systems courses have no relevance to the
humanities.
E. More students major in business because business majors have
the greatest chance of finding a job.
23. The press reports sensationalist stories in order to get people to
pay attention to the news. In order to generate high ratings, reporters
must only report on topics with great scandal that will compel people
to pay attention. The general public is, therefore, uninformed about
basic current events.
The conclusion drawn above is based on the assumption that
A. the press only cares about high ratings.
B. only scandalous topics will generate high ratings.
C. the general public obtains all its news information from news
outlets that generate high ratings.
D. the press should be rewarded for high quality journalism.
E. the press has tried to generate high ratings with quality news,
and has failed.
24. This year, the police department of City X reported a relatively
substantial increase in petty crime. The voters of City X congregated at
a city council meeting to express their concern about this problem. A
leader of a community action group stood up and urged the city council
to rehire the 300 police officers it laid off due to budget cuts in the
previous year. The city council president responded that the increase
was in the actual number of victims who bothered to report petty
crime to the police.
The city council’s statements, if true, best support which of the
following as a conclusion?
A. The previous presence of an extra 300 police officers deterred
criminals from committing petty crime.
B. The laid-off police officers committed petty crime because they
were no longer earning a salary.
C. The budget cuts were a symptom of an economic downturn,
which is usually a cause behind a rise in petty crime.
D. It is possible to determine how many people were victims of petty
crime in a year.
E. The percentage of victims of petty crime in the past year is no
larger than the percentage of victims of petty crime this year.
25. Which of the answer choices most logically completes the
argument given below?
Retail Expert: Companies that manufacture and sell name brand
products often operate under the common misconception that
manufacturing generic products would not be as profitable because
they sell for a lower price. Even though name brand products sell for a
higher price, the cost of production for generic products does not
include any marketing expenses. Thus, the profits generated from the
manufacture and sale of name brand products and generic products
are usually comparable because ____.
A. the market for generic products is large enough and the cost of
production is sufficiently low enough to offset the lower prices
charged to customers
B. the market for name brand products is large enough and the cost
of production is sufficiently low enough to offset the higher
prices charged to customers
C. the respective markets for name brand products and generic
products are about the same size
D. consumer demand will always exist for name brand products
despite their higher prices
E. most companies are savvy enough to manufacture and sell both
name brand and generic products, and they can purposely
balance out those markets
26. Many people complain that standardized tests do not actually
measure knowledge because people simply direct their studies toward
the actual test and not the subject. However, standardized tests do
measure knowledge because test takers have no way of anticipating
the questions on the test and therefore have to study the entire
subject.
The conclusion above is properly drawn if which of the following is
assumed?
A. Standardized tests merely ask particular types of questions on
which certain test takers excel.
B. Test takers must be able to anticipate test questions on any
standardized test in order to direct their studies toward the
actual test and not the subject.
C. If a standardized test has questions that cover the entire
breadth of a subject, then the test actually measures
knowledge.
D. People who direct their studies toward an actual test do not
study an entire subject for the test.
E. Non-standardized tests actually measure knowledge.
27. Many companies offer their employees the option of flex time. Flex
time is the practice of allowing employees to work their allotted
number of hours on whatever schedule that they choose. Flex time
permits employees to work during the most convenient hours for
them, and results in increased productivity because employees are no
longer as distracted by convenience issues.
Any of the following, if true, is a valid reason for a company to offer
flex time to its employees EXCEPT:
A. The majority of workers in any industry report that they often
conduct e-mail correspondence outside of traditional work
hours.
B. Reports show that employees who work from home are often
less productive, and employees who often utilize work from
home options usually opt for flex time instead when their
employers offer it.
C. Employees who work traditional hours often miss time at work
because of health care appointments.
D. The majority of clients report that they prefer to conduct
business over the telephone rather than via e-mail.
E. Employees often exceed their lunch hours because they are
running personal errands that are more convenient to do midday.
28. Vegetarians tend to live longer than people who eat meat. This does
not show that vegetarianism causes people to live longer because
vegetarians as a group tend to exercise more regularly than meat eaters.
Which of the following, if true, most strengthens the argument above?
A. A vegetarian who does not exercise regularly is more likely to start
exercising than a person who eats meat.
B. Among vegetarians, most of those who start to exercise regularly
early in life tend to maintain the practice later in life.
C. Among people who exercise regularly, those who do not eat meat
tend to live as long as those who eat meat.
D. Vegetarianism tends to cause people to engage in other healthy
habits besides regular exercise.
E. A person who eats meat and gives up regular exercise is much
more likely than a vegetarian to never resume regular exercise.
29. The city of Springfield has received a proposal to upgrade its metro
system. An environmental group has argued that increasing the number
of trains and the metro’s area of coverage within the city will result in a
20% increase in rider capacity, reducing the need for people to drive
cars. The city of Springfield should grant the proposal to solve the
problem of traffic congestion on the city roads.
Which of the following, if true, most seriously weakens the argument?
A. Most people in Springfield are unwilling to switch from driving to
public transit.
B. Increasing the metro’s passenger capacity will require the trains to
travel at a slower speed.
C. Most people who drive in Springfield commute to places in the city
that the metro does not currently reach.
D. The proposed upgrade to the metro system will increase air
pollution.
E. The metro system is currently not filled to capacity during the
rush hour commute.
30. President of Fundraising: Donations to our organization have
decreased steadily over the past five years. Many individuals at this
meeting have suggested that the poor economy is the reason. I disagree.
My research has indicated that donors to this cause tend to donate the
same amounts and in the same frequency regardless of their income. I
believe that we need to reexamine our mailings so that we can find a
better way to remind our donors about the important work that our
organization does.
Which of the following, if true, most strongly supports the fundraising
president’s theory?
A. The majority of the donors to this organization have not suffered
financially in the downturned economy.
B. The organization’s mailings used to contain prepaid postage on the
return envelopes for donations, but have not for the past five
years.
C. The organization has decreased its charitable work over the past
five years in order to increase its fundraising effort.
D. The organization has not updated its mailing list for the last seven
years.
E. The majority of the donors are above age 75 and have difficulty
reading smaller print.
31. Dr. Roberts, a dentist, gave a speech to a dental convention in which
she argued that the recent rise in tooth enamel decay in the population
came from a new brand of toothbrush. This brand used a new type of
plastic for the bristles, which Dr. Roberts claimed wears down the
enamel of the teeth over time. Dr. Roberts concluded her remarks by
stating that the consistent high rate of soft drink consumption was not a
significant contributing factor to the recent rise in tooth enamel decay.
Which of the following, if true, most strongly supports the view that the
new brand of toothbrush was responsible for the rise in tooth enamel
decay mentioned above?
A. The brand Dr. Roberts mentioned is the only brand to use the new
type of plastic for the bristles.
B. Consistent heavy consumption of soft drinks does not increase
tooth enamel decay over time provided that patients brush twice a
day.
C. This brand of toothbrush has a 70% market share.
D. The majority of Dr. Roberts’s patients used this brand of
toothbrush for a significant period of time.
E. The company that sold this brand of toothbrush sent hundreds of
thousands of samples to dentists for free giveaways to dental
patients.
32. People between the ages of 40 and 50 purchase more vacations to
Mexico than any other age group. Therefore, the Mexico Tourist Board
should convince hotels and resorts in Mexico to offer more family
friendly packages and services.
Which of the following indicates a flaw in the reasoning above?
A. The author assumes that the majority of people between the ages
of 40 and 50 have children.
B. The author assumes that the majority of people between the ages
of 40 and 50 want family friendly vacations.
C. The author assumes that the majority of people between the ages
of 40 and 50 are married.
D. The author does not supply information about people in other age
groups.
E. The author assumes that the majority of people between the ages
of 40 and 50 can afford a vacation in Mexico.
33. Last year, approximately 1 million smokers quit smoking. This year,
the number of smokers who quit is approximately 3 million. Thus, we can
conclude that the number of smokers who will quit is increasing and that
the number of smokers who will quit next year will be even higher.
Which of the following statements best supports the
conclusion above?
A. The 1 million smokers who quit last year represented the lowest
number in six years.
B. An increase in government regulation regarding health care
usually results in an increase in health care coverage of smoking
cessation programs.
C. This year, a government grant to a program called Light Up No
More brought the number of smokers who quit temporarily up
from the average number of 1 million per year.
D. This year, health insurance companies raised premiums 20% for
customers who smoke, and they plan to institute a 5% increase for
next year.
E. The estimated number of smokers who quit annually is actually
based on a representative sample of the population rather than
the entire population.
34. Production of paper has always been costly, but the tariffs on a box
of paper currently make domestic paper production less expensive.
Therefore, if we want to obtain paper in the cheapest way possible, we
should produce all our paper in our own country.
Which of the following, if true, most seriously weakens the argument
above?
A. Producing paper domestically will create more jobs.
B. Tariffs on boxes of paper are the same flat cost on every box no
matter the size.
C. Companies that produce a product have a high incentive to
control the cost of production.
D. The final price of a product after adding in the tariff is higher than
the impact that production start-up costs have on the final price of
a product.
E. Foreign paper producers can produce more paper overall than a
domestic producer.
35. The retail industry suffers from the problem of high employee
turnover. Since wages remain stagnant for employees who work in
stores, employees have no incentive to remain with the same company
over time. The Namaste Clothing Co. has a much higher retail employee
retention rate than other retail companies. The human resources
department published a report that stated that the company’s success is
due to qualitative factors that the company offers, such as a pension
plan, comp time for salaried employees, casual Fridays, water coolers,
and high-end computers with great tech support.
Which of the following, if true, identifies a flaw in the argument above?
A. The human resources report assumes that employees care about
the qualitative factors listed.
B. The human resources report erroneously lists pension and comp
time for salaried employees as qualitative factors, when they are
actually related to employee compensation.
C. The human resources report lists qualitative factors that would
apply only to company employees who do not work in stores.
D. The human resources report assumes that employees have
remained with the company because of something the company
has offered them.
E. The human resources report did not list all the qualitative
advantages that the company offers its employees.
36. In 2000, women’s clothing retailers reported that 20% of their sales
were for sizes 0–4, 35% of their sales were for sizes 6–12, and 45% of
their sales were for sizes 14–16. In 2010, women’s clothing retailers
reported that those percentages were 25%, 45%, and 30%, respectively.
We can conclude that the average adult woman was the same size in
2010 that she was 10 years ago.
Which of the following, if true, best explains the conclusion above?
A. The amount of clothing purchased in 2010 was greater than the
amount of clothing purchased in 2000.
B. Vanity sizing was much more prevalent in 2000 than in 2010. C
C. A greater percentage of slimmer woman bought clothing in 2010
than in 2000.
D. The number of women who purchased clothing in 2000 was
greater than the number of women who purchased clothing in
2010.
E. Most of the designers who designed clothes in 2010 had not yet
earned their degrees in fashion design in 2000.
37. The average law firm pays entry-level attorneys an annual salary of
$70,000. Ninety-five percent of law school students have jobs upon
graduation. Therefore, of that 95%, the average law student will earn an
annual salary of $70,000.
Which of the following, if true, most seriously undermines the argument
above?
A. Most law students who have jobs upon graduation were hired by
small law firms.
B. The average salary offered by law firms varies by geographical
region.
C. Fewer students enroll in law school today than 10 years ago.
D. The law students who do not have jobs upon graduation did not
pass the bar exam.
E. Over half of law students who have jobs upon graduation were not
hired by law firms.
38. Company XYZ manufactures and sells products that help people quit
smoking. This year, Company XYZ reported an increase in sales overall
and in each individual smoking cessation product. The entire market for
products that help people quit smoking also reported an increase in
sales overall. The product with the highest increase was the patch. The
product with the lowest increase was the gum. We can conclude from
this information that the number of people who have quit smoking has
increased this year.
Which of the following, if true, most seriously calls into question the
explanation above?
A. Most people who quit smoking reported that their greatest
success was with the gum.
B. Most people who quit smoking reported that they did not buy
smoking cessation products from Company XYZ.
C. Most people who quit smoking reported that they relapsed in
approximately 10 months.
D. Most people who quit smoking reported that they bought multiple
smoking cessation products at once.
E. Most people who quit smoking reported that they purchased
whatever smoking cessation product their doctor recommended.
39. The rise of college tuition was widely expected to result in an overall
increase in the total amount that people in this country spent on college
tuition. However, the exact opposite occurred. The rise in tuition
occurred along with a rise in merit scholarships. The result is that
college students in this country actually spend less on college than
previously.
Which if the following would NOT be useful in evaluating the reasoning
above?
A. the total cost of tuition and the monetary amount of the
scholarships
B. the total cost of tuition and the percentage of students who take
advantage of scholarships
C. the total cost of tuition considering scholarships and the cost of
room and board
D. the total cost of tuition considering scholarships and the
breakdown of student attendance at public versus private colleges
E. the total cost of tuition considering scholarships and the number
of students who attend college
40. The state legislature has become concerned with the rise in drunk
driving accidents on state highways. In an effort to curtail this problem,
the legislature raised the driving age from 17 to 18. The governor lauded
this effort, declaring to the press that this new law will drastically reduce
drunk driving accidents.
All the following statements strengthen the argument above EXCEPT:
A. Seventeen-year-old drivers were involved in over half of the drunk
driving accidents on state highways.
B. A famous study reported that the alcoholism rate in minors was
exorbitantly high in this state.
C. A survey of teenagers found that the majority of teenagers did not
believe that alcohol consumption significantly impaired their
driving.
D. A report issued by the department of highway safety asserted that
less driving experience made teenagers slower to react to hazards
on the highway.
E. The state police’s statistics showed that most drunk driving
accidents occurred between 10 P.M. and 6 A.M., and the
department of motor vehicles issued only limited drivers licenses
to teenagers, which permitted driving only during daylight hours.
41. Ten people were hospitalized for severe stomach problems within
one day after drinking Zippo’s Cola. The Zippo Cola Company fervently
argued that its cola drink was not responsible for the stomach problems
and that the company was in no way responsible for anyone’s illness.
Instead, Zippo Cola released a statement to the public that it believed
that the company that makes the soda cans, Carrie’s Canning Inc., used
an ingredient in the metal that made people sick. Margie Smith, one of
the patients, told her doctor that she was convinced that she ate bad
shellfish. All 10 patients ate lunch at the same restaurant that day. The
doctors concluded that both the restaurant and the cola were
responsible for the patients’ illness.
Which of the following, if true, would cast the most serious doubt on
Zippo’s argument?
A. Carrie’s Canning, Inc. also supplied cans from the same can supply
to the Bubbles Orange Soda Company, and no one who drank that
soda became ill.
B. All 10 patients ate shrimp scampi as an appetizer at the restaurant
that day.
C. All 10 patients ate meals at the restaurant that had no overlap ping
ingredients with each other’s meals.
D. Seven of the patients drank regular Zippo Cola and three of the
patients drank Diet Zippo Cola.
E. Margie Smith’s lunch was a pasta dish with mussels. 60
42. The Department of Safe Transportation reported that 73% of all
accidents involve people who are supposed to wear glasses or contact
lenses. We can reduce car accidents if department employees ask
drivers whether they wear glasses or contact lenses, and make a
notation on the driver’s license, every time drivers renew their license.
The argument is flawed because it fails to consider that
A. drivers who are supposed to wear glasses or contact lenses will
wear them while driving.
B. driver’s license renewals will catch a fair number of drivers who
have not notified the Department that they are required to wear
glasses or contact lenses while driving.
C. drivers who are supposed to wear glasses or contact lenses are
not aware of it.
D. a vision test will let drivers know that they need to see an
ophthalmologist.
E. the departments of transportation in other states will not
implement the same policy.
43. University Z created eight intramural sports teams in order to
combat obesity among the student population. The university estimated
that approximately 55% of students were overweight, including 35%
who were obese. Seventy-five percent of students signed up and
participated regularly in the intramural sports teams for the entire year.
At the end of the year, 55% of students remained overweight, and 35%
remained obese.
Which of the following, if true, best explains why the university’s plan
failed?
A. The students who participated in intramural sports continued to
have extremely poor eating habits that exercise could not offset.
B. The students who participated in intramural sports were not
active previously.
C. The students who participated in intramural sports included those
who also participated in official university sports teams.
D. The students who participated in intramural sports possessed
various athletic abilities.
E. The students who participated in intramural sports were
somewhat active previously.
44. Dr. Roberts, a dentist, gave a speech to a dental convention in which
she argued that the recent rise in tooth enamel decay in the population
came from a new brand of toothbrush. This brand used a new type of
plastic for the bristles, which Dr. Roberts claimed wears down the
enamel of the teeth over time. Dr. Roberts added that the consistent
soft drink consumption was not a significant contributing factor to the
recent rise in tooth enamel decay. She then concluded her remarks by
stating that tooth enamel decay in patients will significantly decrease if
every dentist convinces her patients not to use this brand of toothbrush.
The argument is flawed primarily because the author
A. assumed that the cause of increased tooth enamel decay in her
patients is the same as in every other dentist’s patients.
B. assumed that consistent soft drink consumption was not a
significant contributing factor to the recent rise in tooth enamel
decay.
C. assumed that the new type of plastic used for the bristles was the
cause of the rise in tooth enamel decay in the general population.
D. assumed that this brand of toothbrush is the only brand to use the
type of plastic for the bristles that Dr. Roberts claims wears down
tooth enamel over time.
E. assumed that because consistent soft drink consumption was not
a significant contributing factor to tooth enamel decay in her
patients, then it was not in any other dentist’s patients.
45. Television Executive: The largest demographic in this country is
senior citizens. Senior citizens are also more likely to not work than the
rest of the population because they have retired, and therefore spend
more time at home. The majority of senior citizens dislike reality
television. We can therefore conclude that our network should plan
fewer reality TV programs in the future.
Which of the following, if true, would most undermine the television
executive’s argument?
A. Reality television programs cost less to produce than other types
of programs.
B. Senior citizens are not the largest demographic that watches
television.
C. A smaller percentage of senior citizens are currently retired than
in previous generations.
D. The reality TV program called The Golden Years is extremely
popular among senior citizens.
E. Retired senior citizens generally spend their daytime hours
engaged in activities such as gardening, walking, and taking care of
grandchildren.
46. A company has decided to implement a new employee evaluation
program in which pay raises will be determined by an employee’s overall
rating given by his or her supervisor. Upper management concluded that
this new evaluation program would result in pay
raises for the employees with the most meritorious job performance.
Which of the following, if true, does NOT cast doubt on upper
management’s belief that the new program will result in pay raises
based on merit?
A. Supervisors’ judgment of what constitutes “exceeds expectations”
is subjective.
B. All employees who meet revenue goals cannot receive “does not
meet expectations” on their employee evaluations.
C. The three ratings of “does not meet expectations,” “meets
expectations,” and “exceeds expectations” do not provide enough
information to differentiate the various levels of employee job
performance.
D. Supervisors who like their employees tend to rate them higher
than supervisors who dislike their employees.
E. The new employee evaluation program records the information in
a database that the human resources department can access when
processing pay raises.
47. A recent economic downturn has reminded people of the
importance of saving, both for emergencies and for retirement. The
average household saved 5% of its income in the past year, and that
number is predicted to grow over the next 10 years. The result will be
that banks will have more money to lend to customers to start
businesses and buy homes. We can conclude that economic activity will
grow over the next 10 years as a result of this ethic in saving money.
Which of the following would most weaken the argument?
A. People who place a greater percentage of their income into their
savings accounts have been and will be underemployed over the
next 10 years.
B. People who place a greater percentage of their income into their
savings accounts will take out loans to start businesses.
C. People who place a greater percentage of their income into their
savings accounts will also pay down their credit card debt.
D. People who place a greater percentage of their income into their
savings accounts will focus their spending on only necessary home
improvements.
E. People who place a great percentage of their income into their
savings accounts will enable the banks to curtail fee increases.
48. Retail stores that pay their sales associates based on commission pay
their employees wages that vary with the sales dollar amount for which
each employee is responsible. In the retail business last year, stores that
paid their employees on commission showed that worker productivity
was 17% higher than that of their competitors that paid their sales
associates an hourly wage.
If, on the basis of the evidence above, it is argued that paying sales
associates on commission increases worker productivity, which of the
following, if true, would most seriously weaken the argument?
A. Retail stores that pay their sales associates on commission have
experienced that costs other than wages, such as rent, utilities,
and inventory processing, make up an increased proportion of the
total cost of operating each store.
B. Retail stores that pay their sales associates on commission have
more troubling hiring workers because many applicants are
concerned about making little money on days when the stores
have few customers.
C. Sales associates who earn money on commission have paychecks
that are 22% higher than those of sales associates who are paid an
hourly wage.
D. The productivity results cited for sales associates in retail stores
are also true for telemarketers.
E. The majority of customers who shop in retail stores report that
they purchase items without assistance from sales associates.
49. College President: This state’s families actually do not pay less for
tuition at public colleges than private colleges. Each family in this state
pays taxes, a portion of which go to state colleges. When you add the
amount of those taxes with the cost of tuition at a public college, the
cost is actually the same as tuition at a private college. The college
president’s argument is flawed because it fails to consider that
A. the cost of tuition at a private college is higher than the cost of
state taxes paid toward a public college and tuition at a public
college.
B. a portion of taxes that each family in the state pays goes to public
colleges even if no members of that family attend college.
C. many private colleges offer scholarships to students who
demonstrate superior academic achievement.
D. a portion of every family’s taxes in the state goes to public colleges
even if members of that family attend a private college, so the
taxes are not actually part of the cost of tuition.
E. the price of tuition at a public college depends on the state’s
budget.
50. Economist: Large price club type stores will eventually drive small,
independently owned stores out of business. Price clubs can price their
goods much lower per item because they sell products in bulk sizes.
Small stores simply cannot compete with those prices.
Which of the following, if true, does NOT weaken the argument above?
A. Price club stores may be able to offer lower prices, but they only
sell a limited variety of goods, and customers will need to shop
elsewhere to find the remainder of the goods that they want.
B. An increasing majority of customers use public transportation
instead of cars, and live in small apartments rather than houses, so
they cannot transport or store a large number of goods.
C. Many customers desire to support independently owned
businesses, and will shop there rather than at price clubs even if it
means paying higher prices.
D. Independently owned stores accept coupons, and most customers
report that when they factor in coupons, they wind up paying the
same price at independently owned stores as they do in price
clubs.
E. Price club customers report that they earn back the price of their
membership fee within the first two months of purchases.
51. Osteoporosis is a disease that results in a severe weakening of bones.
A diet rich in calcium throughout one’s lifetime is commonly known to be
the best prevention against osteoporosis. Sources of calcium include
legumes, leafy green vegetables, and dairy products. A medical group
devoted to raising the awareness of osteoporosis recommends that
people consume more leafy green vegetables than dairy products in
order to raise their calcium intake.
Which of the following, if true, would most strongly support the position
above?
A. Dairy products typically contain fat, and leafy green vegetables do
not.
B. Leafy green vegetables provide many other health benefits, most
notably a higher iron content.
C. Nutritionists recommend five to eight servings of leafy green
vegetables per day, and only three servings of dairy per day.
D. The body’s absorption rate for calcium in leafy green vegetables is
50%, and for dairy products is 30%.
E. The farming of leafy green vegetables uses fewer environmental
resources than the production of dairy products.
52. Each unit in an apartment complex is supplied with a refrigerator,
stove, microwave, and dishwasher. In order to keep the appliances
functioning, the apartment complex employs a full-time super who is on
call Monday through Friday from 9 A.M. to 5 P.M. Without repairs, the
appliances would eventually stop working and would be expensive to
replace. We can conclude that the apartment complex should continue
to employ the super full-time.
Which of the following, if true, would most seriously weaken the
argument above?
A. Most tenants discover that their appliances are not working
during the evening hours.
B. Apartment complexes that install dishwashers in their units use a
much greater amount of water than those without dishwashers in
their units.
C. Appliances become obsolete in only a few years, which makes
repairing them less practical and more costly then buying new
appliances.
D. Employing a super full-time requires benefits, such as medical
insurance and paid vacation time, in addition to a salary.
E. This apartment complex uses electric stoves, and gas stoves
usually have lower utility bills.
53. The textbook publishing industry reported a consistent annual
increase in e-book sales. For two years, publishers of law textbooks
offered a special promotion, in which the e-book version was offered
free along with the hard copy. Publishers of law textbooks expected this
exposure to e-books to ease law students’ transition away from hard
copies and result in a decline in sales of hard copies in favor of e-books.
Surprisingly, this decline did not occur.
Which of the following would be the most useful to
determine in order to evaluate why the publishers’ promotion did not
achieve the expected results?
A. the average age of law students and their respective familiarity
with the e-book format before the promotion
B. the weight of the average hard copy law textbook
C. the percentage of courses covered by the textbooks with the
promotion
D. the percentage of law students who previously bought e-books
compared to the percentage of undergraduate students who
bought e-books
E. the majority of law students who had open-book exams and took
their exams on their laptop computers, which required an exam
software that blocked student access to all their computer files
during the exam
54. The press has consistently reported stories about the adverse
effects of soy on people with existing thyroid problems, and about the
environmental problems that stem from farming soybeans. A national
health organization stated that fewer people reported consuming soy
products in place of meat and dairy, and an increasing number of people
reported concern that they might be allergic to soy. Therefore, we can
conclude that soy consumption will decrease over the next few years.
Which of the following, if true, would most undermine the conclusion
above?
A. Soy is a common ingredient in processed foods that contain meat.
B. People do not discover a food allergy until after they have had an
adverse reaction to food.
C. Many people suspect that they have a thyroid problem even when
they do not actually have one.
D. People base many of their food choices on the effect they have on
the environment.
E. Meat and dairy products tend to be more expensive than soy
products.
55. Millipoint is a large city that employs many people from the
surrounding suburban communities. Suburban Commute
Transportation Company must expand its services for the increasing
number of people who commute to Millipoint. Suburban Commute
could replace its trains with double-decker trains, add 10% more ferries,
or add 25% more buses. Trains typically spend the least amount of time
in traffic, which is important because they run every 15 minutes. Ferries
have a capacity of 75 people apiece. Buses have a capacity of about 50
people apiece and tend to spend time in traffic. After considering all
options, Suburban Commute decided to replace its trains with
double-decker trains.
Which of the following, if true, could represent the most serious
disadvantage for Suburban Commute’s plan?
A. Two new highways were constructed that lead into Millipoint.
B. Double-decker trains require twice the amount of boarding time
as single-level trains.
C. A competitor has significantly increased its use of the same train
tracks that Suburban Commute uses.
D. Double-decker trains running against rush hour traffic tend to be
nearly empty.
E. Double-decker ferries are available.
56. The Consumer Watchdog Agency reports that 65% of red snapper
sold in grocery stores and restaurants is actually less expensive types of
fish fraudulently labeled. In an effort to clamp down on this fraud, the
agency has created a new consumer confidence stamp that would mark
packages of red snapper as inspected and verified by the Consumer
Watchdog Agency.
Which of the following, if true, provides the strongest reason to expect
that the Consumer Watchdog Agency will succeed in decreasing the
percentage of less expensive types of fish fraudulently sold as red
snapper?
A. Grocery store inventory managers are usually the individuals who
place other fish in the red snapper section of the fresh fish
counter.
B. Restaurant owners have a common practice of passing off other
fish as red snapper.
C. Most customers are unable to tell the difference between red
snapper and other types of fish.
D. The majority of red snapper sold is sold fresh off the boat without
packaging.
E. The fishing industry is interested in curtailing the fraud in red
snapper sales.
57. Potter Shipping, Inc. had a relatively high failure rate in delivering
packages on time. In order to address this problem, the CEO promised
drivers a 10% year-end bonus if they succeeded in delivering 90% of
their packages on time.
All the following, if true, would help to explain why the CEO’s bonus
incentive plan failed to achieve a significantly better on-time delivery
rate EXCEPT
A. highway construction projects caused drivers to spend hours of
time sitting in traffic.
B. most company trucks had faulty GPS units.
C. drivers were responsible to pay for their own fuel, which was
expensive, and faster driving consumed greater quantities of
gasoline.
D. other shipping companies paid their drivers a 15% bonus if they
delivered 90% of their packages on time, and many Potter drivers
wound up leaving their jobs within six months to work for
competitors.
E. most drivers believed the 10% bonus to be a great incentive to
increase on-time deliveries.
58. A group called People for the Preservation of the Environment has
published a damaging report about the pollution caused by Trikas Oil
Refineries. Although not accused of any illegal activity, Trikas has
decided that it must decrease the air pollution it generates in order to
placate public opinion. In order to accomplish this goal, Trikas plans on
upgrading its refineries with the Zeebot Purification System.
The adoption of the Zeebot Purification System would most likely
worsen Trikas Oil Refineries’ problem if
A. the Zeebot Purification System costs a significant amount of
money.
B. the Zeebot Purification System malfunctions 10% of the time.
C. the Zeebot Purification System will take six months to implement.
D. the Zeebot Purification System takes a lot of manpower to
maintain.
E. the Zeebot Purification System results in a significant increase in
water pollution.
59. The government of Woodrock decided five years ago to increase the
number of preventative care medical services covered in the
government’s free health care program for seniors.
Which of the following would be the LEAST important in determining
whether the increased number of preventative care medical services
would be likely to achieve better health for Woodrock’s senior
population?
A. the percentage of seniors who seek preventative care medical
services
B. the success rate of preventative care medical services
C. the cost of preventative care medical services
D. the senior population’s access to facilities that provide
preventative care medical services
E. the percentage of seniors who utilize the government’s free health
care program for seniors
60. In the upcoming elections, the Grow the Economy Now party’s
candidates have proposed a government tax break to individuals and
companies who invest in start-up businesses. The plan would exempt
these individuals and companies from paying any taxes on the profits
from these investments for five years.
All the following would NOT be important to know in determining
whether the tax break would provide an incentive to invest in start-up
businesses EXCEPT
A. the amount of capital required to start a business.
B. the tax bracket of the average individual investor.
C. the most common types of start-up businesses created.
D. the number of years a start-up business requires to earn a profit.
E. the number of candidates in the Grow the Economy Now party
who have a history in investing in start-up businesses.
61. The Lightning Tread Shoe Company, in an effort to boost revenue,
has decided to offer free shipping on its website for all orders over
$22.00. The cost of shipping these orders will reduce the profit by $4.00
per shipment. However, management believes that the savings to
customers will result in more orders.
Which of the following would NOT be important to know in determining
whether this plan will result in an increase in revenue?
A. the current price of packing materials
B. whether competitors of the Lightening Tread Shoe Company also
offer free shipping on orders over $22.00
C. changes in the average price of an order
D. changes in postal rates
E. whether customers purchased the same amount of merchandise
spread out in several smaller orders
62. A citywide survey was conducted in the past year, and revealed that
35% of employees do not know where the fire exits are in their places of
employment. Previously, a private safety company would conduct
annual fire drills in all commercial city buildings that consisted of more
than four floors. In order to increase employee knowledge of the
location of fire exits, the city has arranged to pay fire department
employees a bonus to conduct a second annual fire drill in these
buildings.
Which additional plan, combined with the plan just outlined, will be most
effective in helping the city address the high percentage of employees
who do not know the location of the fire exits in their workplace?
A. hiring the private safety company to conduct annual fire drills in
city buildings with less than four floors
B. paying fire department employees a bonus to conduct a second
annual fire drill in city buildings with less than four floors
C. hiring the private safety company to conduct two annual fire drills
in all city buildings
D. finding a new private safety company to conduct the annual fire
drills
E. threaten to fine companies with over 35% of employees unable to
identify the location of the fire exits
63. Plan: The government of Tulpso was concerned about the high level
of credit card debt that its average citizen carried, and about the high
amount of credit card debt the population carried as a nation. The
government convinced the banks in the credit card industry to offer a
4% lower APR for existing debt to any customer who paid off more than
the monthly interest. The government subsidized the difference in
interest rates.
Result: Many Tulpso citizens still carry the same amount of debt as
before.
Further Information: The annual rate of inflation since the
government-subsidized reduced interest rate on existing credit card
debt has been below 3%, and every Tulpso citizen who carried credit
card debt paid more than the monthly interest of their credit card bills.
In light of the further information, which of the following, if true, does
most to explain the result that followed implementation of the plan?
A. The most recent reduction in APR was only the second of its kind
in the last 20 years.
B. The Tulpso credit card repayment system required citizens to pay
their bills in person.
C. The majority of citizens in Tulspo needed this program to pay
down their credit card debt.
D. The prices of food and other necessities rose to a level that
required citizens to charge similar amounts to what they repaid.
E. The government offered the reduced APR at a time when a large
number of people were poverty-stricken.
64. The population in the country of Xusa has remained stable for the
last 20 years and is expected to remain so. The airline industry has
decided to triple the number of flights offered to current destinations in
the next year.
Which of the following would be most helpful to justify the airline
industry’s plan?
A. The largest segment of the population has reached retirement
age.
B. Businesses have reported a substantial rise in attendance at trade
shows around the nation.
C. The price of airplane fuel has decreased.
D. Companies that produce airplane meals have offered substantially
better wholesale prices to airlines.
E. A famous university study has reported that the percentage of the
population with a fear of flying has dropped significantly over the
last 10 years.
65. The customer service department of the E-Z Movie Rental Co. has
reported a 15% increase in customer service complaints regarding
damaged or unplayable discs in the past year. The CEO announced that
employees responsible for packing the discs into mailer envelopes are
required to perform a new 3-step process to check for scratches and
other visible defects on the discs. Over the next year, the customer
service department reported a 5% increase in customer complaints
regarding damaged or unplayable discs.
All the following, if true, would explain why the new 3-step process
failed to achieve the company goal EXCEPT
A. the paper that the mailer envelopes were made of was a
particularly abrasive type of paper.
B. the warehouse employees did not comply with the new 3-step
process when mailing out discs.
C. most of the customers who complained about damaged or
unplayable discs had damaged those discs themselves.
D. the majority of the discs’ defects were visible defects.
E. damaged or unplayable discs were not taken out of circulation.
66. The Whigham Insurance Company offers All Risk insurance to
hotels, resorts, and other large structures. All Risk insurance covers
everything that is not listed in the exclusions. There has been a recent
increase in earthquakes and Whigham is concerned that earthquake
coverage will be too expensive to maintain. However, Whigham would
still prefer to avoid listing earthquakes in the list of exclusions for the
future if possible.
Which of the following strategies would be most likely to minimize
Company X’s losses on the policies?
A. insuring only those customers whose buildings did not experience
earthquake damage in the past 20 years
B. insuring only those customers who cannot obtain earthquake
coverage through another insurance provider
C. expanding its marketing campaign to attract customers with
buildings far away from fault lines
D. insuring only those customers who can afford to pay for
earthquake coverage for their buildings
E. insuring other disasters such as tornadoes, hurricanes, and
monsoons, in the hopes of maintaining customer loyalty
67. A company that manufactures laundry detergent has decided to
switch from petroleum-based ingredients to plant-based ingredients.
The company believes that the switch will decrease its environmental
footprint and address frequent customer complaints that the detergent
tends to cause the color in clothes to fade quickly.
Which of the following, if true, most helps to provide a justification for
the company’s planned switch?
A. The cost of petroleum-based ingredients and plant-based
ingredients is the same.
B. The company plans to move its manufacturing overseas.
C. The annual sales of laundry detergent have decreased over the
past two years.
D. The company will continue to manufacture stain remover pens
that have stronger ingredients.
E. Consumer spending on higher-quality clothing is anticipated to
grow over the next 10 years.
68. Currently, the market for ice cream is dominated by dairy-based ice
cream. To increase its revenue, the Simply Soy marketing department
developed a new campaign for its line of soy-based ice cream flavors.
The marketing team convinced the product development team that it
should amplify the food coloring used in all the flavors, and the
marketing team would release ads with pictures of the ice cream
designed by a food stylist.
What condition will make the marketing department’s plan more likely
to increase revenue?
A. The food coloring used in the Simply Soy flavors consists of all
natural ingredients.
B. The number of people who are lactose intolerant has increased.
C. People who buy ice cream are not bothered by food that contains
artificial colors.
D. The public in general is gravitating toward more plant-based
foods.
E. People tend to base their grocery shopping decisions based on
how appealing the products look.
69. Both universities and the government are concerned about the low
level of student achievement in math and science. In addition, the
government is also concerned about the rising cost of university tuition.
A university suggested that private enterprise could locate some of their
research in university laboratories, and provide tuition assistance to
promising students who aid in the research.
Which of the following, if true, would be most useful to know in
evaluating the university’s suggested plan?
A. the size of the student applicant pool for the research program
B. the science and math aptitude of the student applicant pool for
the research program
C. the value of incentives that universities would offer private
enterprise to conduct research in university laboratories
D. the number of students who would major in math and science as a
result of the plan
E. the type of research that private enterprise would conduct in
university laboratories
70. Construction Company Y always used power drills that plug into an
electrical socket. Power-Z has manufactured a new type of
battery-powered drill that uses so little energy that it is much cheaper
to use. Thrilled that the price of both kinds of drills is the same,
Construction Company Y has decided to replace all its drills with the
new Power-Z drill, and estimates that the replacement costs will pay off
within three years.
Which of the following, if it occurred, would constitute a disadvantage
for Construction Company Y of the plan described above?
A. construction Company Y would need to buy 350 new drills
B. an increase in the market demand for construction work
C. other companies that manufacture power tools introduced similar
drills at the same price
D. a significant increase in the price of batteries in the next year
E. a shortage of industrial-sized extension cords in hardware stores
71. A national long distance track coach created a new cross-training
program for the team, which includes swimming, weight training,
sprinting staggered with jogging, and yoga. The coach has stated to the
press that the new cross-training program will result in an Olympic
podium sweep by the nation’s team.
Which of the following, if true, could present the most serious
disadvantage for the coach’s cross-training program?
A. The yoga exercises would enhance the runners’ breathing
capabilities under physical stress.
B. None of the national team’s competitors have considered this type
of cross-training program.
C. The cross-training program would lead to improvement in some of
the runners on the team, but runners on other teams would likely
have trouble with it.
D. The program requires additional athletic gear, which is very
expensive.
E. The weight training would add muscle to the runners, which would
result in a slight weight gain that would slow down their times.
72. Winston University had many cases of meningitis the previous year.
The Director of Residence Life presented a plan to prevent this from
happening again this year. All university students covered by the student
health plan would be required to obtain a meningitis vaccine before
moving into the dormitories.
The answer to which of the following questions would be most
important in determining whether implementing the proposal would be
likely to achieve the desired result?
A. the percentage of Winston University students who are covered
by the student health plan
B. the percentage of Winston University students who live in the
dormitories and are covered by the student health plan
C. the percentage of Winston University students who live in the
dormitories
D. the percentage of Winston University students who live off
campus
E. the percentage of Winston University students who did not
receive a meningitis vaccine the previous year
73. A non-governmental organization raised money to donate
thousands of anti-malarial mosquito nets to a country in which malaria is
a serious public health threat. The organization also operated a chain of
free clinics in that country. The organization transported the nets in
April of that year. Four months later, the president of the organization
announced that the severe decline in malaria outbreaks would continue
throughout the remainder of the year, and adjusted the clinic budget by
slashing its orders for quinine, the treatment for malaria, in half.
Which of the following, if true, would most undermine the president of
the organization’s decision to order half as much quinine for the year?
A. Quinine is used to treat a few other uncommon ailments.
B. The chain of clinics treats on average only 40% of the country’s
population.
C. The wet season, during which malaria is prevalent, begins in
October and ends in March.
D. The percentage of the population that received the anti malarial
mosquito nets is unclear.
E. Other clinics operated by different organizations ordered the
same amount of quinine.
74. Foreign language skills are vital in today’s international economy.
Studies have shown that the earlier a student begins to learn a language,
the more proficient that student will become with that language. The
school district of M wants to increase students' foreign language skills.
The superintendent of the M school district concluded that requiring all
students to begin studies in a foreign language in fourth grade instead of
seventh grade will result in a greater number of students who are
proficient in a foreign language.
Which of the following, if true, would cast the most doubt on the success
of the superintendent’s plan?
A. The majority of students who study a foreign language in the
school district already speak a foreign language at home.
B. The instruction time for foreign language study will be significantly
less for the earlier grades than for grades 7 and above.
C. Students will still have a choice between only two languages
D. Foreign language study for fourth graders will take some
instruction time away from social studies.
E. Most elementary school teachers in school district M do not have
training in a foreign language.
75. Trial courts in the state’s civil division have found that jurors pay less
attention in civil trials than they do in criminal trials, which jurors find to
be more interesting. In order to maintain the attention of jurors, the
county bar association proposed that all civil litigators use color and/or
interactive visual aids.
Which of the following, if true, provides the strongest reason to expect
that the bar association’s plan will succeed in getting jurors to pay
attention during civil trials?
A. Jurors will find the color and/or interactive visual aids as
interesting as criminal trials.
B. Jurors will find the color and/or interactive visual aids interesting
enough to hold their attention.
C. Judges will allow the civil litigators to use color and/or interactive
visual aids.
D. Civil litigators will actually want to use color and/or interactive
visual aids.
E. Criminal litigators will not start to use color and/or interactive
visual aids.
76. The Everglades has an ecological problem because of its population
of Burmese pythons, which are not a native species of the area and were
likely introduced by people who dumped them when they were no
longer desirable as pets. In order to address this problem, a local
ecological group created a procedure for safely capturing the pythons.
Which of the following, if true, would provide the most support for the
conclusion that the ecological group will capture most or all of the
Burmese pythons in the Everglades?
A. The ecological group has a group of dogs trained to sniff out the
location of the Burmese pythons.
B. The local government has increased the fines for people caught
releasing animals into the Everglades.
C. The Burmese python is one of several snakes in the Everglades
that are not native to the area.
D. Burmese pythons are not venomous.
E. Other environmental groups have also created plans to reduce the
Burmese python population in the Everglades.
77. A state has used the speed limit of 65 miles per hour on its highways
over the last several years. A study has shown that many drivers pulled
over for speeding on these highways were not driving at an unsafe
speed. In order to lower the number of drivers pulled over for speeding
who were not driving at dangerous speeds, the state has decided to
increase the speed limit on its highways to 70 miles per hour.
Which of the following would be most useful in evaluating the state’s
plan to lower the number of drivers pulled over for speeding who were
not driving at dangerous speeds?
A. the number of drivers who will continue to drive at speeds greater
than 70 miles per hour
B. the percentage of drivers who were pulled over for speeding as
opposed to the percentage of drivers who were pulled over for
other reasons
C. the number of the state’s voters who believe that speeds of 70
miles per hour and under are not unsafe
D. the number of drivers who were pulled over for driving at speeds
between 66 miles per hour and 70 miles per hour
E. the number of drivers on the state’s highways during rush hour
78. The Bridgeview shopping mall’s customer service department has
reported many complaints from customers about a shortage of parking
in the parking deck. The Bridgeview mall drafted a plan in which it would
construct two additional levels to its parking deck, and would charge
each retailer in the mall a $4,000 fee to help pay for it, to which the
retailers all agreed.
Which of the following, if true, would be most useful to evaluate the
benefit to the retailers for agreeing to pay the fee in order to enable
more customers to shop at the mall?
A. the percentage of mall customers that drive to the mall, as
opposed to taking other forms of transportation
B. whether the additional number of parking spaces will alleviate the
parking shortage
C. the actual number of complaints to customer service about a
shortage of parking spaces
D. the cost of the plan to the Bridgeview mall after the retailers each
paid their fee
E. whether the $4,000 fee that each retailer would pay toward the
additional two levels in the parking deck would actually pay off in
sales revenue
79. A municipality is concerned about the number of car accidents that
occurred during the winter. The mayor suggested that it consider buying
and staffing snowplows in order to alleviate the problem. The
municipality purchased four snowplows, and offered a $15 per hour
wage to applicants to drive the snowplows when needed.
Which of the following would be most useful in determining whether the
municipality’s plan to decrease car accidents during the winter season
will actually succeed?
A. whether job applicants will drive the snow plows for $15 per hour
B. whether most winter car accidents are a result of snow
C. whether most car accidents in the municipality occur during
winter
D. whether four snowplows will be enough
E. whether most cars in the municipality have snow tires 86
80. Kitchen, Kitchens, Kitchens, Inc.! is a kitchen appliance company that
is looking to increase sales for stainless steel appliances. The company
designed its new line of appliances to be more energy efficient.
Advertisements will show the total of the annual savings in energy costs
plus the average government tax rebate to homeowners for increasing
the energy efficiency of their homes.
Which of the following, if true, raises the most serious doubt regarding
the company’s expected success from its new advertising campaign?
A. Competitors plan on highlighting the appearance of the appliances
in their advertisements.
B. The savings generated by energy savings and the tax rebate are
less than the cost of the new appliances.
C. Stainless steel appliances always have a higher price than
appliances in basic colors.
D. Most customers want to buy their kitchen appliances in matching
sets.
E. The savings generated by energy efficient appliances and the tax
rebate are negligible.
81. Many universities have elected to turn their undergraduate
programs into 5-year programs, in order to incorporate mandatory
internships for credit. Although students will have an additional year of
study, they will gain valuable working experience that will look great on
their resumes in this recession and this experience will be worth the cost
of the additional year’s tuition.
Which of the following, if true, provides the most support for the
potential success of attracting undergraduate applicants to 5- year
undergraduate programs?
A. Most students take out only a small quantity of money in student
loans, and find their monthly payments to be affordable.
B. The recession has meant that employers have their pick of
students, and prefer those with experience.
C. Students have the opportunity to intern full-time during their
summers.
D. Most of the internships are offered in big cities that students
regard to be fun and exciting.
E. About half of undergraduate institutions are planning on offering
these 5-year programs.
82. The We Care Insurance Company wants to expand by offering new
insurance products. One of the members of the product development
team proposed that the company offer pet insurance. The team agreed
that it was a good idea, until the member who pitched the idea shared an
industry study, which stated that pet insurance clients tend to avoid
policies that raise the annual premium based on the number of claims
submitted. Since pet insurance cannot legally drop a pet’s coverage once
the pet becomes ill or significantly injured, the product development
team is worried that pet insurance will not be profitable after all.
Which of the following strategies would most likely enable the We Care
Insurance Company to offer pet insurance?
A. capping the amount of covered services per pet at the cost of an
annual checkup and shots
B. require pet owners to bring their pets only to veterinary practices
that participate in the We Care Insurance Company’s plan
C. insure only those customers who are wealthy enough to afford
many veterinary services for their pets
D. raise premiums based on the age of the pets
E. insure only pets with no history of medical problems
Answers
1. B.
Choice b identifies the strategy in Peter’s counterargument. Peter
points out that Louis did not consider that the financial industry’s losses
in profits and jobs will result in profit and job increases for others.
Choice a is incorrect because Peter does not offer additional evidence to
support Louis’s argument. Choice c is incorrect because Peter does not
argue that the regulations on derivatives will have a negative effect on
the national economy. He argues that the increase in profits and jobs in
other industries will compensate for the loss in profits and jobs in the
finance industry. Choice d is incorrect because Peter does not challenge
Louis’s facts; he supplements them. Choice e is incorrect because Peter
does not challenge the relevance of Louis’s facts.
2. d.
Choice d is correct because the cost of repairing the cell towers does not
need to be lower to justify the cost savings of this plan. Those repairs
must cost more than the cost of this plan for the cell towers. Choices a,
b, c, and e are incorrect because they all mention items that should cost
less than the repairs to the cell towers during the previous year.
3. e.
This is a problem that you should attempt to sketch out. Try to connect
the names to make a chain. Katherine has a higher GPA than Pamela and
Adrienne. Shane has a higher GPA than Madelyn, who has a higher GPA
than Adrienne.
P&A<K
A<M<S
Choice e is correct because Adrienne cannot have a higher GPA than
Shane. Choices a and b are incorrect because the argument does not tell
you whether Katherine or Madelyn has a higher GPA. Choice c is
incorrect because the argument does not tell you whether Pamela or
Adrienne has a higher GPA. Choice d is incorrect because Pamela could
have a higher GPA than Madelyn.
4. a.
The argument shows that Sharon did not inherit the cottage because
Claire McGinnis did not own the cottage when she died. Choice a is
correct because we do not know whether Claire would have left the
cottage to her niece, even though Claire left Sharon the rest of her
property. Choices b and e are incorrect because whether Humphrey
Monroe wanted Sharon to inherit the cottage is irrelevant to the
argument’s conclusion. Choice c is incorrect because whether Sharon
wanted to inherit the cottage is irrelevant. Choice d is incorrect because
the assumption is about who Claire wanted to inherit the cottage, not
who Claire did not want to inherit it.
5. A.
Choice a is correct because it is possible that the employees in State A
each work such a low number of hours per week and that the employees
in State B work such a high number that the total number of hours paid
for is actually higher in State B. Choice b is
incorrect because there is not necessarily a correlation between the
number of employees and the number of restaurants since many
employees could work very few hours. Choice c is incorrect because the
statement would actually contradict the argument’s conclusion. Choice
d is incorrect because there is no indication that state law has anything
to do with why employees in State A are paid a higher wage. Choice e is
incorrect because the argument does not say anything about what might
happen in the future.
6. B.
Choice b is correct because in order for buying diamonds to cost 40%
more in Luvania, the tax on purchases by foreigners in Oretania and the
cost of transportation to Oretania must be 40% less than the cost of
buying diamonds in Luvania. Choice a is incorrect because the ease of
the ground breaking under the force of a pickaxe in Oretania is merely a
plausible explanation for the cost difference, but there are also other
possible reasons. Choice c is incorrect because if the transportation cost
is more than 40%, buying diamonds in Oretania would be more
expensive. Choice d is incorrect because the cost of diamond mining is
merely a possible explanation for the cost difference. Choice e is
incorrect because there is no evidence about retail jobs in jewelry stores
in the passage, and the passage also indicated that people would
actually be less likely to buy diamonds in Luvania.
7. D.
Choice d is correct because fixing the filtration system at the point of
groundwater collection will not change the excess chlorine content if
the problem starts later in the process. Choice a is incorrect because
whether all faulty water filtration systems allow excess chlorine into
drinking water is irrelevant. Choices b and e are incorrect because they
introduce assumptions that would disprove the argument. Choice c is
incorrect because it states a fact listed in the argument.
8. D.
Choice d is correct because whether survey respondents felt superficial
in admitting that their physical appearance was important to them does
not explain why 100% admitted that it was, yet 15% of respondents did
not cite concern over their physical appearance as a reason for quitting
smoking. Choice a is incorrect because it reflects the possible reason
that respondents saw a connection between their physical appearance
and cigarette smoking, and did not care enough about it to quit smoking.
Choice b is incorrect because it indicates the possible reason that
respondents saw a positive correlation between their physical
appearance and cigarette smoking. Choices c and e are incorrect
because they reflect the possible reason that respondents did not see
much, if any, connection between their physical appearance and
cigarette smoking.
9. C.
This problem appears tricky, but it is actually pretty simple if you
remember not to make any assumptions about the listed apartment
traits and their respective values. The prompt tells you that a higher
rental price results from a higher floor, more sunlight, and/or a greater
number of bedrooms. What we do not know is how various
combinations compare to each other, so it is important not to assume
anything, such as a greater number of bedrooms having a higher value
than greater sunlight. What you want to look for is the one answer
choice that clearly violates the following setup:
Cathy—6th floor
David—4th floor
Krystal—1st floor, $$$$ (highest rent)
Oscar—basement floor, $ (lowest rent)
Choice c is correct because it violates the setup. If Krystal lives in a
1-bedroom, David’s apartment has the most sunlight, and Cathy lives in
a 3-bedroom, then how can Krystal pay the highest rent? She cannot
because her apartment would not meet any of the conditions that could
plausibly price her rent the highest. In choice a, Krystal could pay the
highest rent because no one is listed as having an apartment with more
bedrooms than hers. Choice b appears tricky because Krystal has a
smaller apartment than Cathy, but Krystal’s apartment might receive far
more sunlight than the others. Choice d is incorrect because the
conditions provided support the fact that Oscar pays less rent than the
other two tenants listed in the answer choice. Choice e is incorrect
because even though Cathy’s top floor apartment gets the most
sunlight, the additional two bedrooms in Krystal’s apartment provide a
reason that she pays the highest rent.
10. E.
Choice e is correct because a higher membership in weight loss
programs fails to explain why women who spent more money on
exercise clothes and went to the gym more often did not lose more
weight than other women. Choices a, b, c, and d are all reasons why the
women who spent more money on exercise clothes, and went to the gym
more often, did not lose more weight than those who spent less money
on their exercise clothes.
11. E.
Choice e is correct because a larger percentage of patients diagnosed
with bacterial infections would not justify a decrease in the percentage
of patients prescribed antibiotics. Choices a, b, c, and d are all reasons
that would explain a drop in the percentage of patients prescribed
antibiotics.
12. A.
Choice a is correct because digital products earn a higher percentage of
profits than the percentage they make up of sales. Choice b is incorrect
because books earn the greater percentage of profits. Choice c is
incorrect because the passage says nothing about growth. Choice d is
incorrect because the passage does not say anything about the number
of products in each group. Choice e is incorrect because books earn the
same percentage of profits as digital products and products in other
mediums together.
13. B.
Choice b is correct because the cost of travel for high-level employees
would need to be less than the savings generated by near-shoring in
order to justify the practice. Choice a is incorrect because the prompt
does not say anything about businesses wanting to convince current
employees in cities with a higher cost of living to relocate. Choice c is
incorrect because the prompt does not indicate that businesses would
have to pay for any cost of client travel. Furthermore, even if the clients
traveled out of the cities with a higher cost of living, that would actually
further justify moving employees out of those locations, including
higher-level employees who deal with clients. Choice d is incorrect
because although it provides some support for near-shoring, there is not
enough information about financial incentives to provide a comparison
to the savings generated by near-shoring. Choice e is incorrect because
the clients staying put does not provide an incentive for near-shoring.
14. C
This question is asking for the MOST important item to ascertain, so be
careful with answer choices that list items that are relevant but are not
the most important. Choice c is correct because water has to be
available to the drought victims in order to give them relief. Choice a is
incorrect because the exact number of relief organizations involved is
irrelevant. Choices b, d, and e all list items that are important to know
but are not the most important, because without water, the rest of these
items will have no impact on the drought relief plan.
15. B.
This is a formal logic argument, so you should try to sketch it out and
make connections between the people. Courtney is the most commonly
mentioned name, so try to see where everyone falls in relation to her.
Courtney has less money than Marnie, Alexandra, and Josie. Alexandra
has less money than
Danielle. Therefore:
C . . . M/A/J
C...A...D
Choice b is correct because Danielle cannot have less money than
Courtney, since Alexandra has more money than Courtney, and
Alexandra has less money than Danielle. The other choices are all
possible.
16. C.
Choice c is correct because Sophia pointed out that Taylor failed to
consider that registration is a benefit after the purchase of securities,
not before. Choice a is incorrect because Sophia does not provide
evidence that people actually read the securities paperwork. Choice b is
incorrect because Taylor’s argument is not about the volume of paper
that registration generates, and Sophia’s response merely indicates that
buyers of securities will have a remedy against the parties who
misrepresented them. Choice d is incorrect because Taylor’s argument is
about the volume of information that registration generates, not about
the financial literacy of people who buy securities. Choice e is incorrect
because Sophia does not suggest that all securities be registered; she
only states the advantages of registration for those securities that
already must be registered.
17. B.
Look for the choice that is required for the argument to be true. Choice
b is correct because both options together cannot result in greater fuel
efficiency than only the use of a lighter metal in the car frames. Choice a
is incorrect because ABC’s options for a lighter metal would not change
the argument that the use of a lighter metal in the frames would be the
most that ABC could do to increase fuel efficiency. Choice c is incorrect
because the cars’ fuel efficiency can still be increased even if it already is
good. Choice d is incorrect because what ABC wants to do and the most
that ABC can do are two different things. Choice e is incorrect because
cost is not relevant to ABC’s best effort to increase fuel efficiency.
18. E.
Choice e is correct because a lower market demand and higher
distribution in the overall industry would be reasons for Crunchy
Organics’ profits to decline. Choice a is incorrect because decreased
wages would enable the company to earn a greater profit. Choice b is
incorrect because lower utility bills and a tax refund would result in a
greater profit. Choice c is incorrect because a larger olive crop means a
greater supply of olives, and rainfall does not cost the company money.
Choice d is incorrect because a feature about the company as an ethical
business could reasonably result in more consumers buying olive oil
specifically from this company.
19. C.
Choice c is correct because women like to see how a product will look
on them, which is a realistic image. Choice a is incorrect because the
affluence of women in cosmetics advertisements is not relevant to the
passage. Choice b is incorrect because the passage provides no
information that most women actually shop for daytime cosmetics
rather than makeup more appropriate for evening. Choice d is incorrect
because glamorous is not the same as realistic. Choice e is incorrect
because the passage does not indicate that women are dissatisfied with
the quality of cosmetics featured in advertisements.
20. E.
Choice e is correct because the passage does not explicitly say that the
installation of the machines in town that give a 5-cent refund per item
will motivate people to recycle. Furthermore, the passage does not
indicate whether any resulting increase in recycling would be enough to
save those jobs. Choice a is incorrect because the township’s trash plant
is irrelevant and the 5-cent refund will not pay for any costs because
residents will pocket it. Choice b is incorrect because the passage does
not question the recycling plant’s capacity for additional materials.
Choices c and d are incorrect because the 5-cent refund would go to
residents, not for the cost of the machines.
21. D.
Choice d is correct because the public health expert provides
information about population size that negates the reporter’s claim.
Choice a is incorrect because the issue is the number of cases of the
virus in relation to the size of the entire population. Choice b is incorrect
because the public health expert does not single out particular people
who are infected with the Q Virus. Choice c is incorrect because the
argument never alleges an incremental increase, only an increase.
Choice e is incorrect because the expert does not question information
about the people, only about the group as a whole and its relation to the
entire population.
22. A.
Choice a is correct because the argument draws a conclusion based on
the percentage of students enrolled in each college. Choices b and d are
incorrect because the argument does not say anything about the
qualities of computer information systems courses. Choice c is incorrect
because the argument does not say anything about course
requirements. Choice e is incorrect because the job market is not
relevant to the argument.
23. C.
Choice c is correct because people obtaining news from sources that do
not have high ratings would undermine the argument’s conclusion.
Choice a is incorrect because the conclusion does not rely on what the
press cares about. Choice b is incorrect because the argument already
made this statement. Choice d is incorrect because what the press
should do has no role in the argument. Choice e is incorrect because
whether the press has failed to obtain high ratings with quality news is
not an assumption necessary for the conclusion that people will not
obtain news elsewhere.
24. E.
Choice e is correct because the city council is arguing that crime did not
increase. Do not let the answer’s use of percentages fool you. The key is
that the percentages did not change year to year. Choice a is incorrect
because this conclusion follows from the argument by the community
action group leader. Choice b is incorrect because the passage says
nothing about the laid-off police officers committing petty crime. Choice
c is incorrect because the city council president is not stating causes for
an increase in petty crime; she is stating that no increase in petty crime
has occurred. Choice d is incorrect because the city council’s claim that
only the number of reported petty crimes has increased does not
indicate that it is possible to actually find out how many people were
actually victims in a year.
25. A.
Choice a is correct because a large market and low production cost are
both valid criteria for offsetting a lower price for generic products.
Choice b is incorrect because a large market and low production cost for
name brand products are not needed to offset higher prices. Higher
prices would naturally generate higher profits, and therefore do not
need to be offset at all. Choice c is incorrect because the passage does
not provide any information to link similar profits with a similar market
size. Choice d is incorrect because the passage says nothing about either
of the markets in the future. Choice e is incorrect because the argument
states that companies that manufacture and sell name brand products
are often under this misconception about the difference in their profits,
which indicates that most companies are not savvy enough to
manufacture and sell both brand name and generic products.
26. B.
Choice b is correct because the argument assumes a connection
between anticipating test questions and gearing study toward the
question types. Choice a is incorrect because the argument is about how
people study for standardized tests, not about whether certain people
excel on those tests due to question types. Choice c is incorrect because
the argument does not say anything about tests covering the entire
breadth of a subject. Choice d is incorrect because the argument
concludes that people have to study the entire subject for tests. Choice
e is incorrect because the argument does not mention non-standardized
tests.
27. D.
Choice d is correct because telephone correspondence requires that
clients are at work at the same time as employees at the companies that
serve clients. Although employees could schedule telephone calls during
flex time, this answer provides a
circumstance in which flex time is not beneficial, even if it is not
detrimental here. Choice a is incorrect because it shows that e mail
correspondence occurs at all hours, which could actually make flex time
beneficial. Choice b is incorrect because it shows that flex time can make
a particular group of employees more productive. Choices c and e are
incorrect because they show negative results of employees working
regular hours, which flex time can fix.
28. C.
The argument establishes a correlation between regular exercise and
longevity, and states that vegetarianism is not the cause of greater
longevity. Choice c is correct because it strengthens the argument that
there is no causal connection between vegetarianism and longevity by
showing that both meat eaters and vegetarians have the same longevity
when they exercise regularly. Choice a is incorrect because it states an
association between vegetarianism and the likelihood of exercising
regularly, which would weaken the argument. Choice b is incorrect
because it does not negate the causal connection between
vegetarianism and longevity. Choice d is incorrect because it promotes
an association between vegetarianism and other healthy habits that fails
to weaken a causal connection between vegetarianism and exercising
regularly. Choice e is incorrect because it merely states a correlation
between meat eating and unhealthy habits.
29. A.
Choice a is correct because if most people in Springfield are unwilling to
switch from their cars to public transit, the upgrade will not significantly
reduce the number of drivers on the streets. Choice b is incorrect
because the speed of the metro trains has no impact on the argument.
Choice c is incorrect because the current limited reach of the metro
supports the argument that increasing its area of coverage would entice
drivers to use public transit instead. Choice d is incorrect because
although an increase in air pollution may counteract additional reasons
that the
environmental group may have to support the proposed upgrade for the
metro, air pollution does not have a relationship in this argument with
getting drivers to switch to public transit. Choice e hints that people do
not desire to ride public transit, but does not weaken any argument that
people are unwilling to switch.
30. E.
The question is asking for the choice that bests supports the president
of fundraising’s theory that donors are giving less because they are less
aware of the work that the organization does. Choice e is correct
because smaller print, if used in the mailings, could cause the donors to
be unable to read about the
organization’s work, and therefore feel less inclined to donate. Choice a
is incorrect because the fundraising president stated that the economic
downturn did not have a relationship to donation activity. Choice b is
tricky, but it is incorrect because although the prepaid postage on the
donation envelopes has to do with the mailings, it does not inform the
donors about the organization’s work. Choices c and d are incorrect
because they provide different reasons for the decrease in donations.
31. C.
Choice c is correct because it shows that the majority of people who
brush their teeth used this brand of toothbrush. Choice a is incorrect
because it only establishes that no other brands used the same type of
plastic for the bristles, not that a significant number of people used this
brand of toothbrush, which would be necessary to show the brand’s
effect on the tooth enamel of the population. Choice b is incorrect
because it does not address the brand of toothbrush under discussion,
and whether people brushed twice a day is outside of the scope of the
dentist’s theory. Choice d is incorrect because the majority of Dr.
Roberts’ patients do not provide a basis for making a statement about
the general population. Choice e is incorrect because the toothbrush
company’s promotional activities did not necessarily result in a
significant portion of the population using those toothbrushes.
32. B.
Choice b is correct because the author makes a faulty assumption that
people between the ages of 40 and 50 want a particular style of
vacation, when the argument does not provide any information to that
effect. Choice a is a tricky one, but it is incorrect because the author
does not assume that people between the ages of 40 and 50 have
children, but that they want to travel with their families as opposed to
other people. Choice c is incorrect because the author does not assume
that people between the ages of 40 and 50 have a family that includes a
spouse, but that they want to travel with their family. Choice d is
incorrect because the argument tells us that people between ages 40
and 50 purchase the most vacations to Mexico. We do not need to know
information about other age groups. Choice e is incorrect because
whether the majority of people in a particular age group can afford a
vacation in Mexico is outside of the scope of the argument.
33. D.
Choice d is correct because it provides a reason that more smokers
chose to quit this year. A higher cost for the following year would
support the conclusion that even more smokers will be motivated to quit
the following year. Choice a is incorrect because the previous year
having the lowest number does not tell us whether the following year
will have a higher number than the current year. Choice b is incorrect
because the answer does not tell you whether an increase in
government regulation has actually occurred or will occur in the future.
Choice c is incorrect because it merely provides a possible reason for
the increase in the number of smokers who quit this year. It does not
provide any information about whether that grant will be available for
the following year. Choice e is incorrect because it does not provide any
information about the following year.
34. B.
Choice b is correct because fewer larger boxes of paper will generate
lower tariffs and might wind up being less expensive than domestic
paper production. Choice a is incorrect because job creation is not
relevant to cost reduction here. Choices c and d are incorrect because
they actually would strengthen the argument. Choice e is incorrect
because the amount of paper produced overall does not necessarily
have relevance here as a foreign paper producer likely produces and
sells paper to several countries.
35. C.
Choice c is correct because the human resources report fails to
recognize that the qualitative benefits it lists generally do not pertain to
store employees. Choice a is incorrect because there is no information in
the prompt that indicates that company employees do not care about
the qualitative factors listed. Choice b is incorrect because whether
pension and comp time for salaried employees relates to employee
compensation does not matter; the point is whether these items actually
help retain store employees. Choice d is incorrect because the argument
states that the company has done something to retain employees.
Choice e is incorrect because additional qualitative factors that may
have retained employees are irrelevant to the flaw in this argument.
36. C.
Choice c is correct because it explains why a greater percentage of
clothing in smaller sizes purchased in 2010 does not show that the
average size of adult women has changed in 10 years. Choices a and d
are incorrect because variations in the amount of clothing purchased or
in the number of adult women who purchased clothing does not explain
the discrepancy between percentages of various sizes purchased and a
lack of change in the average adult woman’s clothing size. Choice b is
incorrect because it supports the opposite conclusion than the one
posited. Vanity sizing would have to be more prevalent in 2010, not
2000, to explain this conclusion. This is the type of answer choice
designed to trip you up because you may have assumed that the correct
answer would be vanity sizing before you looked at the answer choices.
Choice e is incorrect because a change in the majority of people who
designed clothes merely hints at the vanity sizing issue mentioned in
choice b, but does not actually indicate any change in the way the
designers designed clothes. Remember that you are looking for the best
answer.
37. E.
Choice e is correct because if most law students who have jobs upon
graduation were not hired by law firms, then the average annual salary
of $70,000 provides no information about their salaries. Choice a is
incorrect because the argument provides no information about salary in
relation to the size of the law firm. Choice b is incorrect because
geographical region is outside of the scope of the argument. Choice c is
incorrect because the argument makes a statement about the
percentage of law students, not the total number. Choice d is incorrect
because the argument does not draw a conclusion about law students
who did not pass the bar exam.
38. D.
Choice d is correct because multiple purchases by each person who
attempted to quit smoking would negate the association between the
number of the smoking cessation products purchased and the number of
people who quit smoking. Choice a is incorrect because the fact that
people reported that they had more success with the gum may imply
that they might have failed with the other products, which would cast
some doubt on the argument, but this statement is not as conclusive as
choice d. Choice b is incorrect because Company XYZ may be one of
many companies that manufacture and sell smoking cessation products.
Choice c is a tricky one, but be careful. Someone who quit smoking and
then relapsed still quit smoking, albeit temporarily, so choice c is
incorrect. Choice e is incorrect because doctor recommendations are
irrelevant in this argument.
39. D.
Choice d is correct because the breakdown of student attendance at
public versus private colleges is irrelevant; the actual total in overall
tuition is what matters. Choice a is incorrect because theoretically a
majority of students could have scholarships, but they could be for such
low amounts that they barely offset the cost of tuition. Choice b is
incorrect because even if scholarships offer a large discount on college
tuition, a low percentage of students who have them could result in
scholarships having a minimal effect on the expense of tuition. Choice c
is incorrect because the conclusion is about college, not college tuition
specifically. Therefore, room and board is a relevant cost of college.
Choice e is incorrect because the number of students who attend
college affects overall college expenditures.
40. E.
Choice e is correct because teenagers could only legally drive during
daylight hours and the most drunk driving accidents occurred during
late hours. Choices a and d are statements that would support the state
legislature’s belief that drivers under age 18 significantly contribute to
or are less capable of avoiding drunk driving accidents. Choices b and c
are statements about teenagers and alcohol that would support an
inference that teenagers are responsible for a percentage of drunk
driving accidents.
41. A.
Remember to read the question closely, because this one asks you for
the answer that would cast the most serious doubt on Zippo’s argument,
NOT the doctors’ argument. Choice a is correct because it would cast
doubt on the argument that an ingredient in the cans caused the illness.
Choices b, c, and e are incorrect because they are about what the
patients ate, which does not undermine Zippo’s argument. Choice d is
incorrect because whether the patients drank regular or diet Zippo Cola
does not matter: they all got sick.
42. C.
Choice c is correct because if most people who need to wear glasses or
contact lenses are unaware of that fact, then they obviously will not
notify the department of transportation when they renew their driver’s
licenses, and the plan will not reduce accidents. Choices a and b are
incorrect because they are assumptions of the argument, not factors
that the argument failed to consider. Choice d is incorrect because the
prompt does not mention a vision test. Choice e is incorrect the prompt
does not mention anything about out-of-state drivers in relation to car
accidents.
43. A.
Choice a is correct because it provides a reason why increased athletic
activity among students did not result in any weight loss among the
student body. Choice b is incorrect because it provides a reason why the
program should have succeeded. Choice c is incorrect because there is
no information regarding what percentage of students who participated
in the intramural sports were also on official university athletic teams.
Choice d is incorrect because athletic ability does not reveal anything
about student weight before and after participation in the intramural
sports. Choice e is incorrect because students being somewhat active
before participating in intramural sports merely hints that the increased
physical activity was insufficient to result in weight loss.
44. D.
Choice d is correct because Dr. Roberts’ conclusion is that the rate of
tooth enamel decay will drop if patients stop using this brand of
toothbrush. However, the problem is actually the type of plastic in the
bristle. If other brands used the same plastic in the bristles, patients
would still suffer the same rate of tooth enamel decay. Choices a and e
are incorrect because they merely assume a connection between Dr.
Roberts’ patients and dental patients in general. Choices b and c are
incorrect because these claims by Dr. Roberts do not address the brand
of the toothbrush in the conclusion.
45. B.
Choice b is correct because the television executive assumes that since
senior citizens are the largest demographic of the population and spend
more time at home due to retirement, they watch the most television. If
another age group watched the most television, then the executive
should gear programming toward that age group. Choice a is incorrect
because the mention of cost is not directly tied to viewer popularity,
which is the subject of the argument. Choice c is incorrect because it
merely states that fewer senior citizens spend time at home than
previously, not that they necessarily watch less television than other
groups. Choice d is incorrect because it simply mentions one reality TV
show popular among seniors; theoretically, that group may not watch
any other reality programs. Choice e is incorrect because it only
provides reasons that hint that senior citizens do not spend that much
time watching television, and those reasons only apply to daytime.
46. E.
Choice e is correct because database storage shows a feature of the
program that is efficient for human resources to use. Choices a and d are
incorrect because supervisors can allow personal likes or dislikes to
color their evaluations of employees. Choice b is incorrect because
employees who meet revenue goals may be deficient in other job
responsibilities. Choice c is incorrect because differences in merit may
vary more than at three levels.
47. A.
Choice a is correct because if the people who are saving are
underemployed, their income is probably lower than it was previously
and an increased percentage of income saved could actually result in
less money to spend, which will not increase economic activity. Choice b
is incorrect because loans to start businesses may generate more
economic activity. Choice c is incorrect because the public paying down
its credit card debt would bring more money to the banks, which would
facilitate increased economic activity. Choice d is incorrect because
spending only on necessary home improvements does not indicate how
much or little economic activity is generated by that activity. Choice e is
incorrect because bank fees are not relevant to the conclusion.
48. E.
Choice e is correct because if the majority of customers purchase items
without help from sales associates, then the connection between
increased worker productivity and sales rates is broken. Choice a is
incorrect because wages becoming a smaller proportion of total costs
does not weaken the argument. The other costs could have grown faster
than the cost of wages. Choice b is incorrect because hiring is not
relevant to the argument. Choice c is incorrect because sales associates
who work on commission having higher paychecks supports the
argument that they are more productive. Choice d is incorrect because
other industries are not relevant.
49. D.
Choice d is correct because the money that state residents pay in their
taxes is not actually a cost of tuition, and families pay tax even if their
children attend private colleges. Choice a is incorrect because the
college president states that the cost of tuition at a private college
equals the cost of tuition at a public college plus the cost of taxes paid
toward public colleges. Choice b is incorrect because it only explains
that everyone pays the taxes. Choice c is incorrect because scholarships
at private colleges could result in private colleges costing less. Choice e
is incorrect because the state’s budget is not relevant in this argument
to the cost that state families pay toward tuition at a public college.
50. E.
Take note that this question asks for which answer does NOT weaken
the argument, NOT which answer strengthens the argument. Therefore,
cross off each answer that weakens the argument and choose the one
that remains. Choice a is incorrect because customers will shop at other
stores for goods if price clubs do not sell them. Choice b is incorrect
because customers who cannot transport or store bulk sizes of goods
are unlikely to shop at price clubs. Choice c is incorrect because it states
that some customers will not shop at price clubs despite the low prices.
Choice d is incorrect because it negates the claim that price club prices,
the reason that customers would presumably choose to shop there over
other stores, are not actually lower than those in other stores. Choice e
is correct because it does not address the price comparison between the
two stores.
51. D.
Choice d is correct because it provides justification for the medical
group’s recommendation of one food item that contains calcium over
another in order to lower one’s chances of osteoporosis. Choice a is
incorrect because the argument does not say that fat intake has any
effect regarding osteoporosis. Choice b is incorrect because the
argument does not concern itself with other health issues. Choice c is
incorrect because the medical group’s argument is not about the general
daily diet, only about calcium intake. Choice e is incorrect because the
effect on the environment is not relevant to the argument.
52. C.
Choice c is correct because if the installation of new appliances is less
expensive than repairing the current ones, the reason for the complex to
employ a full-time super is weakened. Choice a is incorrect because the
time of day that tenants discover problems with the appliances is not
important; what is important is the cost for the complex to have the
appliances fixed. Choice b is incorrect because the amount of water the
appliances use tells us nothing about whether the complex should
continue to have the appliances fixed. Although you might infer that
more water used means a higher cost, the answer does not provide any
information to infer that the cost from greater water use would negate
the savings from fixing the appliances. Choice d is incorrect because the
answer does not provide any information indicating that the cost of the
super’s benefits would negate the savings of having the super fix the
appliances. Choice e is incorrect because it does not compare the
expense of utility bills with the expense of paying the super. In addition,
tenants commonly pay utility bills, which would mean that the utility bill
savings suggested here are irrelevant to the argument.
53. E.
Choice e is correct because it provides a reason why law students would
not buy the e-book. If a large percentage of students had open-book
exams and took them on the computer using software that blocked
access to computer files during the exam, these students would not have
access to the e-book during the exam; they would therefore have a
reason to continue purchasing hard copies. Choice a is incorrect because
the age and familiarity with e-books before the promotion would not
explain why the promotion did not succeed in making more students
prefer e-books after the promotion. Choice b is incorrect because the
weight of a hard copy textbook, if heavy, does not explain why the
promotion failed. Choice c is incorrect because regardless of the
percentage of courses covered by the promotion, some decrease in hard
copies purchased would be expected. Choice d is incorrect because a
comparison to undergraduate students is not relevant to why a decline
in the purchase of hard copies of law textbooks did not occur as a result
of the promotion.
54. A.
Choice a is correct because the argument states reasons why people
plan not to consume soy and fewer people plan to consume soy in place
of meat and dairy; if soy is abundant in processed foods that contain
meat, people may inadvertently consume an increased amount of soy.
Choice b is incorrect because the increasing number of people
concerned that they might be allergic to soy would presumably have
already had an adverse reaction to something they ate, believe that soy
was the culprit, and plan on avoiding soy in the future. Choice c is
incorrect because suspicion of a thyroid problem, even if not well
founded, is enough to convince people to avoid soy when they believe
that soy will exacerbate the problem. Choice d is incorrect because it
would actually strengthen the argument. The passage states that
soybean farming causes environmental problems. Choice e is incorrect
because although a price difference is a reason that people would
consume more soy, price difference fails to negate one of the reasons
the passage listed for less future consumption of soy.
55. C.
Choice c is correct because the advantage of the trains was that they do
not spend much time in traffic. A competitor significantly increasing its
use of the same tracks could result in train traffic problems. Choice a is
incorrect because it merely presents an advantage of the bus option, not
a disadvantage of the train option. The answer also does not indicate
whether the construction of two new highways will necessarily alleviate
traffic on the road. Choice b is incorrect because the passage does not
say how long trains take to board in the first place, and the amount of
boarding time is unlikely to cause a problem with a train system that
runs at 15- minute intervals. Choice d is incorrect because the low use of
the trains for trips in small demand is not a factor in Suburban
Commute’s capacity needs for peak trips. Choice e is incorrect because
it merely presents an advantage of the ferry option.
56. E.
Choice e is correct because the fishing industry’s desire to decrease
fraudulent red snapper sales could help the Consumer Watchdog
Agency. Choice a is incorrect because if grocery store inventory
managers are placing other fish in the red snapper section of the fresh
fish counter, the fish is being sold unpackaged and customers will not
see the stamp. Choice b is incorrect because restaurants serve prepared,
and not packaged, food, so customers would not see the stamp. Choice c
is incorrect because customers failing to discern red snapper from other
fish will not help solve the fraud problem. Choice d is incorrect because
if most red snapper is sold without packaging, the stamp verification
would not appear to be helpful.
57. E.
This is an EXCEPT question, so note that you are looking for the answer
choice that does NOT explain why the plan failed. Choice e is correct
because the drivers finding the 10% bonus to be an incentive would not
explain why they failed to deliver 90% of their packages on time. Choice
a is incorrect because traffic problems are a plausible reason that delays
were beyond the drivers’ control. Choice b is incorrect because faulty
GPS units could result in drivers arriving at the wrong locations and
therefore delivering packages late. Choice c is incorrect because if fuel
was expensive enough, the cost of driving fast could outweigh the
benefit of the 10% bonus. Choice d is incorrect because if drivers could
earn more money at another company and therefore the majority left
Potter Shipping after six months, the drivers would not remain for a year
to collect the bonus, thereby eliminating the incentive.
58. E.
Choice e is correct because Trikas wants to decrease air pollution in
order to placate public opinion, and a significant increase in water
pollution is likely the negative effect that their air pollution problem has
on public opinion. Choice a is incorrect because cost is irrelevant to the
immediate goals of Trikas listed in the prompt. Choice b is incorrect
because the system may still significantly reduce air pollution even if it
malfunctions 10% of the time. Choice c is incorrect because the prompt
does not provide a deadline for Trikas to meet its goals. Choice d is
incorrect because the manpower required to maintain the system is
irrelevant to the immediate goals of Trikas.
59. C.
The question asks you to identify which answer choice is LEAST
important in determining whether the government’s new coverage of
preventative care medical services will increase senior health. Choice c
is correct because since this program is free to seniors, the cost of the
services does not matter. Choice a is incorrect because if the percentage
of seniors who seek preventative care is very low, then the program
would not improve senior health. Choice b is incorrect because the
program would not improve senior health if the preventative services
detected health conditions but early detection provided no advantage in
treating those conditions. Choice d is incorrect because if a great
percentage of seniors do not have access to preventative care, the
program will not improve senior health. Choice e is incorrect because if
only a low percentage of seniors use the government’s program, then
the program will not significantly impact the health of the senior
population.
60. D.
The question is asking you to identify the answer choice that would be
helpful in determining whether this plan would provide an incentive to
invest in start-up businesses. Choice d is correct because if the average
start-up business did not turn a profit for the first five years, this tax
break would not be advantageous for the average investor in a start-up
business. Choice a is incorrect because the amount of capital needed
does not tell us anything about the tax break. Choice b is incorrect
because the average tax bracket of an individual investor has no impact
on this proposed tax break. Choice c is incorrect because the type of
business is irrelevant to the tax break. Choice e is incorrect because the
number of Grow the Economy Now party candidates who would benefit
from this tax break has no connection to whether the tax break would
serve as an incentive to investors in general.
61. A.
The question asks you to identify which answer choice would NOT be
relevant in determining whether the free shipping offer would result in
an increase in revenue. Choice a is correct because the price of packing
materials would not matter because the prompt tells you the expected
loss in profits. Only changes in this price would matter, and the answer
choice does not mention changes. Choice b is relevant because
competitors providing the same offer would not make shopping at The
Lightening Tread Shoe Company particularly advantageous for
customers. Choice c is relevant because if the average price of an order
dropped by over $4.00, the company could lose money. Choice d is
relevant because higher postal rates could mean a loss in profits greater
than $4.00 in the future. Choice e is relevant because customers
splitting their purchases into several smaller orders could result in the
company losing more money without gaining in merchandise purchases.
62. B.
The prompt does not explain whether the majority of the 35% work in
buildings with more than four stories. Therefore, the additional plan
should include those buildings. Choice b is correct because it includes
the smaller commercial buildings AND it addresses the possibility that
the private safety company might be doing an inadequate job in the
buildings it currently covers. Choices a and c are incorrect because they
do not address the possibility that the private safety company’s job
performance is inadequate. Choice d is incorrect because it does not
address the possibility that commercial buildings smaller than four
floors might be the source of the problem. Choice e is incorrect because
the effectiveness of fines is unclear, especially as the answer does not
provide a monetary amount for the fine. For example, a $1 fine is
unlikely to provide an incentive.
63. D.
Choice d is correct because it explains that citizens in the repayment
plan wound up not paying off any debt because they were charging at
the same rate. Choice a is incorrect because the current repayment plan
could have enabled citizens to reduce their credit card debt no matter
how many similar programs the government had offered in the past.
Choice b is incorrect because the inconvenience of paying bills in person
does not explain why people who participated in the program did not
reduce their credit card debt. Choice c is incorrect because the need for
this program does not explain why it did not succeed. Choice e is
incorrect because the program could still be expected to reduce debt
even though people were living in poverty.
64. B.
The question is asking you to find a valid reason for the airline industry’s
decision to triple its flights. Choice b is correct because an increase in
business attendance at trade shows around the nation would be a
reason to expect an increase in travel. Choice a is incorrect because
even if the percentage of retirees has increased, they will not necessarily
travel more. Choices c and d are incorrect because they only state that
an area of operating costs has decreased, not that there is a greater
demand for air travel. Choice e is incorrect because there is not
necessarily a correlation between a lower percentage of people who
fear flying and a greater demand for air travel.
65. D.
The question asks you to identify the answer choice that would not
explain why the new 3-step process failed. Choice d is correct. Note that
the prompt states that the 3-step process checked for “scratches and
other visible defects.” If the majority of the defects were visible, then the
process would have caused employees to discover them. Choice a is a
plausible explanation because it explains that the mailer envelopes are
scratching the discs when the warehouse employees slide the discs into
them, which would occur after the employees inspected the discs.
Choices b and e would clearly explain why the process failed. Choice c
would explain the failure because the damage to the discs would have
occurred after the company mailed them out.
66. C.
Choice c is correct because the strategy would increase the customer
pool to include customers who are less likely to file claims for
earthquake damage. Choice a is incorrect because since earthquakes are
on the rise, customers with buildings that have not experienced
earthquake damage in the past 20 years may be more likely to
experience it in the future. Choice b is incorrect because customers who
cannot obtain earthquake coverage through another provider probably
cannot do so because their likelihood of experiencing earthquake
damage in the future is high. Choice d is incorrect because people who
can afford earthquake coverage will pay for it; the problem will arise
when too many claims are filed in the future and the company cannot
afford to pay all of them. Choice e is incorrect because insuring other
disasters is irrelevant to the problem of maintaining coverage for
earthquakes.
67. E.
Choice e is correct because if people are buying higher quality clothing,
they are more likely to want to preserve the dye in them. Choice a is
incorrect because although the same cost is not an argument against the
switch, it does not show that the switch will have an additional
advantage. Choice b is incorrect because moving manufacturing
overseas does not provide an additional justification regarding the
switch to plant-based ingredients. Choice c is incorrect because a
decrease in consumer spending for laundry detergent overall indicates a
declining consumer demand for the product. Choice d is incorrect
because stain remover pens are not in the scope of the prompt’s
discussion of the company’s plan.
68. E.
Choice e is correct because whether the plan will increase revenue
depends on whether the ads and the ice cream’s enhanced colors entice
people to buy the product. Choice a is incorrect because natural
ingredients are irrelevant to the marketing department’s plan. Choices b
and d are incorrect because the marketing department’s plan does not
have anything to do with the product’s soy base. Choice c is incorrect
because the prompt does not reveal whether the food coloring uses
artificial ingredients. Assuming that it does, and even if people do not
mind artificial ingredients, that fact will not motivate them to buy the
product.
69. C.
Choice c is correct because the university’s suggested plan would
require private enterprise to have a motive to conduct research in
university laboratories and to offer tuition assistance to students.
Choice a is incorrect because the size of the applicant pool is not the
most relevant consideration, as private enterprise would have to first
participate in the program. Choice b is incorrect because the suggested
plan does not indicate that students have to possess great achievement
levels in math and science in order to participate. Choice d is incorrect
because the universities and the government are concerned about
achievement in math and science, not necessarily in the number of
majors. Choice e is incorrect because the type of research is not
particularly relevant to whether this suggested plan would succeed.
70. D.
Choice d is correct because an increase in the price of batteries could
negate the savings generated by using the new drills. Choice a is
incorrect because the number of drills that Construction Company Y
would need to buy does not matter. Choice b is incorrect because an
increase in the demand for construction work would bring Construction
Company Y more business. Choice c is incorrect because other
companies releasing a similar product at the same price would not
negate the construction company’s savings. Choice e is incorrect
because a shortage of industrial-sized electrical cords would not matter
since the new drills are battery operated.
71. E.
Choice e is correct because the cross-training program would be
disadvantageous if it resulted in slower times. Choice a is incorrect
because it is an advantage of the program. Choice b is incorrect because
what the team’s competitors have not considered does not mean the
program is not advantageous for this team. Choice c is incorrect because
all that matters is the advantages of the runners for this team. Choice d
is incorrect because the price of the athletic gear does not take away
from the goal of the program.
72. B.
Choice b is correct because the prompt stated that the plan only
addresses students who are covered by the student health plan.
Furthermore, the prompt stated that students would be required to
have the vaccine before moving into the dormitories, which would mean
that students living off campus would not be affected by the plan.
Therefore, choice a is incorrect. Choice c is incorrect because it leaves
out the variable of students covered by the student health plan. Choice
d is incorrect because it does not address the issue that the vaccination
plan only applies to students on the student health plan. Choice e is
incorrect because it fails to address the issues of students on the
student health plan and students who live in the dormitories.
73. C.
Choice c is correct because if the wet season has not yet begun, the
president of the organization does not yet have confirmation that the
mosquito nets have actually reduced cases of malaria. Choice a is
incorrect because a small need for quinine for other purposes does not
negate cutting orders in half when the large public health threat that
requires quinine is perceived to have abated. Choice b is incorrect
because the perceived success of the anti-malarial mosquito nets would
presumably have an effect on the 40% of the population that seeks
treatment at the
organization’s clinics. Choice d is incorrect because it merely suggests
that the mosquito nets’ success might not be widespread, but does not
give any reason to affirmatively see a failure. Choice e is incorrect
because other clinics simply may not have known about this
organization’s donation of anti-malarial mosquito nets.
74. B.
Choice b is correct because less time given to foreign language study for
younger students will offset the benefits that are supposed to be gained
by teaching students a foreign language at a younger age. Choice a is
incorrect because students who already speak a foreign language at
home do not necessarily study the same foreign language at school.
Choice c is incorrect because a choice limited between two languages is
not relevant to proficiency in a foreign language. Choice d is incorrect
because taking instruction time away from social studies will not
negatively affect proficiency in a foreign language. Choice e is incorrect
because the superintendent’s plan does not necessarily indicate that the
elementary school teachers will not receive training or that they must
be the ones to teach a foreign language.
75. B.
Choice b is correct because the plan requires that jurors will actually
find the visual aids interesting in order for the plan to succeed. Choice a
is incorrect because the plan does not require that jurors find civil trials
as interesting as criminal trials. Choice c is incorrect because judges
allowing the visual aids is a requirement for the plan to work, not
something that would actually show that jurors find the aids interesting.
Choice d is incorrect because the litigators wanting to use the aids will
not make the jurors find them interesting. Choice e is incorrect because
what criminal litigators do is irrelevant to the plan.
76. A.
Choice a is correct because it explains how the members of the
ecological group will find the Burmese pythons. Choice b is incorrect
because increased fines for releasing animals does not explain whether
the ecological group’s capturing procedure will succeed. Choice c is
incorrect because other non-native snakes are not part of the group’s
objective. Choice d is incorrect because the passage already states that
the procedure for capture is safe, so whether the Burmese python is
venomous is not an issue. Choice e is incorrect because the plan is only
about this ecological group, not others.
77. D.
Note that the state’s speed limit plan assumes that a speed of 70 miles
per hour is safe. Choice d is correct because the number of drivers who
were pulled over under the old speed limit would have to decrease
under the new speed limit in order for the plan to work. Choice a is
incorrect because the number of drivers who will continue to drive over
70 miles per hour has no bearing on the state’s change in the speed limit;
what matters is the number of drivers who were driving 70 miles per
hour or less. Furthermore, the state’s plan is about drivers pulled over
for driving at that speed, not drivers who drove that fast in general.
Choice b is incorrect because the state’s plan has nothing to do with
other reasons drivers were pulled over for besides speeding. Choice c is
incorrect because what the voters think about the speed limit is not
relevant. Choice e is incorrect because the passage does not provide
enough information to make any associations between the density of
traffic during rush hour and the speed limit.
78. E.
Choice e is correct because each retailer would want to earn the $4,000
back in sales revenue for the fee to be worth paying. Choice a is
incorrect because the percentage of mall customers who drive does not
necessarily indicate a correlation between increased revenue and
increased parking. Choice b is incorrect because the purpose of the
parking plan for the retailers is to increase revenue, which could occur
even if a parking shortage continued after the two new levels in the
garage were built. Choice c is incorrect because the number of
complaints about the parking shortage only indicates the problem, not
how severe the problem may be. Many customers who were unable to
find parking may not have bothered to complain to the mall’s customer
service department. Choice d is incorrect because the cost to the mall
does not affect the retailers’ decision to pay $4,000 apiece.
79. B.
Choice b is correct because the municipality’s plan assumes that a large
cause of car accidents during the winter is snow. Choice a is incorrect
because whether applicants will drive the snowplows for the offered
wage does not address the plan’s assumption that snow is the primary
cause of the car accidents. Choice c is incorrect because the plan only
addresses car accidents that occur during the winter season. Choice d is
incorrect because the number of snowplows does not address the plan’s
assumption that snow is the major cause of these accidents. Choice e is
incorrect because snow tires are out of the plan’s scope.
80. E.
Choice e is correct because the company’s advertising plan is premised
on the assumption that the savings generated will be sufficient to attract
customers to this line of appliances. Choice a is incorrect because the
advertising campaigns of competitors are not relevant. Choice b is
incorrect because it assumes that customers will only buy these
appliances if the savings will actually cancel out the cost of the
appliances. Choice c is incorrect because the price of appliances in basic
colors does not matter regarding this advertising campaign. Choice d is
incorrect because customers would presumably still see savings in
buying a matching set of these appliances.
81. B.
Choice b is correct because the passage states that the benefit to
students of the 5-year programs is job experience that employers find
valuable. In a recession, this reason is even more compelling. Choice a is
incorrect because it says that most students find their current loans to
be affordable, but provides no indication that the expense of a fifth year
of tuition would still be affordable. Choice c is incorrect because other
time opportunities for internship experience are a reason that students
would less likely find 5-year programs attractive. Choice d is incorrect
because it merely provides an additional reason that students would be
attracted to 5-year programs. It does not address job experience. Choice
e is incorrect because the fact that half of undergraduate institutions
plan on offering these programs does not indicate that students would
necessarily enroll in all the available slots in these programs, which if
true, could support an inference that students not in these programs
would be at a competitive disadvantage. Even with this inference, this
statement does not address the value of job experience and its
relationship to the cost of the fifth year of tuition.
82. D.
Choice d is correct because age provides a mechanism for raising
premiums, and since older pets are likely to need more medical care, the
premiums are more likely to cover the cost of submitted claims. Choice a
is incorrect because capping the price of pet insurance at the cost of an
annual checkup and shots simply does not make sense for either
customers or the insurance company. If the insurance cost equals or is
greater than the costs of the checkup and shots, customers have no
reason to buy it because they might as well pay for those costs out of
pocket. If the insurance cost is less than the cost of the checkup and
shots, the company would lose money on the policies. Choice b is
incorrect because the plan does not indicate that participating
veterinary practices would somehow make offering the policies
affordable. Choice c is incorrect because people who are wealthy
enough to afford many veterinary services would still only buy insurance
if the price provided an advantage, and they would still submit claims.
Choice e is incorrect because the problem is that the insurance company
has to insure the pets after their medical problems are revealed.